Sie sind auf Seite 1von 109

Homework Problem Solutions for MSE

156/256:
Solar Cells, Fuel Cells and Batteries:
Materials for the Energy Solution
Bruce M. Clemens
Stanford University
Autumn 2014

c
Copyright 2014
Bruce M. Clemens

ii

Contents
1 Energy and Solar Spectrum

2 Semiconductor Physics

13

3 Photovoltaic Devices

27

4 Solar Cells

47

5 Electrochemistry and Batteries

77

iii

iv

Chapter 1
Energy and Solar Spectrum
1. At noon on a sunny day, there is a solar power density of 1 kW/m2 impinging on a
solar panel that is 1 meter by 1.6 meters. To simplify the problem we assume here
that the light is all the same wavelength = 732 nm.
(a) Determine the number of photons per time incident on the panel.
(b) Determine the current generated if the panel has 100% efficiency (every incident photon generates one electron that contributes to current).
Solution:
The easiest way to solve this problem is to think about it conceptually.
(a) You are given the Solar Irradiance, which is in units of Power/Area. This can
be converted to # photons/time by determining the area of the panel and the
energy of one photon of 732 nm light

power
energy
=
area
time area
energy
1
# photons
Area
=
time area
energy/photon
time

(1.1)

Since it is a rectangular panel, the area is L W = 1 m 1.6 m = 1.6 m2 .


The photon energy can be found from
1

E=

hc
6.626 1034 J s 3 108 m/s
=
= 2.716 1019 J/photon
9

732 10 m

If we plug everything back into our original equation (Eq. 1.1) we get

1000 W/m2 1.6 m2

1
2.716

1019

J/photon

= 5.89 1021 photons/s

(b) Current can be thought of as the quantity of charge that passes a single point
in a wire per unit time. From part (a) we know the # of photons per second
hitting the panel. The problem states that one electron is generated per
photon. Thus, we can determine the # of electrons generated in the material
per second. Since we know the charge of an electron we can determine the
charge per second generated in the material, which is equal to the current.

electrons
photons
=
s
s
electrons
charge
charge

=
= current
s
electron
s
5.89 1021 electrons 1.6 1019 C

= 944 C/s = 944 Amps


s
electron
2. (a) What is the power density of the sun shining on a solar cell lying flat on
the ground at the National Renewable Energy Laboratory in Golden, CO
on September 23, 2007 (the autumnal equinox) at noon? Assume the solar
radiation to be parallel rays with a power density perpendicular to the rays of
1.35kW/m2 and neglect losses due to atmospheric effects.
(b) What is the total power incident on a 1 m wide solar cell running from the
north pole to the south pole on the same day?
Solution:
(a) Here we are neglecting all atmospheric contributions, so all we are concerned
with is the effective power density on the cell due to the incident radiation
2

P0 = 1.35 kW/m2 . We must take into account the angle of the sun on the
cell, which lies flat on the ground. The latitude of Golden, CO is 39.74 N.
According to Figure 1.1 we can see that the power density is given by the
quantity
a/c P0 = P0 cos

N
Golden, CO: 39.74 deg N

Solar Cell
Incident Sunlight

Figure 1.1: Schematic showing the incident sunlight and orientation of the solar panel
So for our latitude, the power density is given by
1.35 kW/m2 cos 39.74 = 1.04 kW/m2
(b) To find the total power generated by a 1 m wide cell stretching from the N to
S poles of the earth at the same time as the main question, we consult Figure
1.2 below. To set up the problem, we consider an infinitesimal area element
of the solar cell, given by da in the illustration. This area element is 1 m wide
and length dl along the long direction of the cell. We can write the length dl in
terms of the radius of the earth, Re , and the latitude angle, , as dl = Re d so
that da = Re wd. Then we can write the power collected by the infinitesimal
area as the power density at that times the area of the infinitesimal area as
dP = P ()da = P0 cos Re wd
where w is the width of the cell, here 1 m.
3

dl
Re

Figure 1.2: Schematic showing the setup for the pole-to-pole cell calculation
Next, to find the total power being collected we integrate up all the dP s from
pole to pole as

Z
Power =

P0 Re w cos d = P0 Re w [sin sin()]

dP =

= P0 Re w (1 (1))
= 2P0 Re w
and using values for all the parameters given above we get that
Power = 2 1.35 kW/m2 6.37 106 m 1 m = 17.2 GW
Shortcut
Another way to solve the problem is to realize that the earth only collects flux
from the sun over a linear distance of the diameter of the earth (2Re ) and so the
cell running from pole to pole, since atmospheric effects are neglected, collects the
equivalent power of a cell in space that is 1 m wide by 2Re long. Thus, the solution
to the problem is just
Power = 2P0 Re w = 2 1.35 kW/m2 6.37 106 m = 17.2 GW
4

3. A 3.1 kW peak system that was recently installed on a campus house cost a total of
$30,000 after rebates from the government. Graph the cost per KWh of the energy
generated by this system as a function of the lifetime of the solar panels used (from
5 years to 50 years). Note: State any assumptions you made for this calculation
and please use a computer for all graphs
Solution:
Since solar cells on a private/commercial rooftop require a significant initial investment, the cost per KWh of the electricity the cells generate is directly dependent
on the total electricity the cells generate over their lifetime:
Initial Cost
30, 000
day
year
Lifetime
Cost
=
=

kWh
Total Energy Generated
3.1 kWp 6 hours 365 days
x years
If we vary the number of years in a lifetime, the curve seen in figure 1.3 can be
generated.

Figure 1.3: Cost/kWh vs. Lifetime


4. Cadmium Telluride (CdTe) is considered a promising 2nd generation solar cell
technology, due to its high efficiency, low cost, and long-term stability. However,
5

one of the main issues with CdTe solar cells is the total amount of Tellurium (Te)
available on the earth. According to the United States Geological Survey, the global
production of Te was estimated to be 135 metric tons in 2007, and the world reserve
of Te is estimated to be about 48000 metric tons.
(a) If we use all the Te produced in 2007 to make 2m thick CdTe solar cell
modules with 10.5% efficiency, calculate the total power that can be generated
under one sun (1 kW/m2 ) by these panels.
(b) Now calculate the total power that can be generated under one sun if we
consume all the Te in the worlds reserves to make these panels. (The density
of CdTe is 5.85 g/cm3 )
(c) Suppose that tellurium costs $215/kg. What is the cost of tellurium used in
1kW of CdTe solar cells?
(d) CdTe cell manufacturer First Solar claims that its production cost is $0.76/W.
Compare this to the cost of the tellurium it uses.
Solution:
(a) To calculate the total power it is possible to generate with one years worth
of Te production we must first know how much Te is in a solar cell per unit
area.
Te weight
= (CdTe Density) (Te weight fraction in CdTe) (Thickness)
unit area
The weight fraction of Te in CdTe is:
Te weight fraction in CdTe fTe
MTe
MTe + MCd
127.6 g/mol
=
127.6 g/mol + 112.411 g/mol
= 0.53
=

where M is the molecular mass.


The annual power available is then:
Annual power =

(Annual global production)(Power density from the sun)(Efficiency)


(CdTe density)(Te weight fraction in CdTe)(Thickness)
6

Annual power =

(135 1000 1000 g) (1 kW/m2 ) (10.5%)


= 2.28 GW
(5.85 106 g/m3 ) (0.53) (2 106 m)

(b) The total power that could be generated if you use all the Te available can be
calculated by replacing the annual global production by the total Te in the
earths crust:

Total power =

Total power =

(Total production)(Power density from the sun)(Efficiency)


(CdTe density)(Te weight fraction in CdTe)(Thickness)
(48000 1000 1000 g) (1 kW/m2 ) (10.5%)
= 0.81 TW
(5.85 106 g/m3 ) (0.53) (2 106 m)

(c) To calculate the cost of Te per kW peak solar production we just take the
volume of Te needed to make one kW times the density times the weight
fraction times the price:
($215/kg) (2 106 m) (5.85 103 kg/m3 )fTe
$Te
=
kW solar power
1 kW/m2 10.5%
= $12.74/kW
(d) The cost of the Te works out to about $0.013 per watt, and so is a pretty
small part of the total cost of the cell
5. For this problem you will need to use a spreadsheet program such as Excel or
Numbers. Download the file SolarSpectrum from the class Coursework webpage (it is in the HW folder). This file comes from NREL and contains the ASTM
G173-03 AM1.5 solar spectrum downloaded from url rredc.nrel.gov/solar/spectra/am1.5/.
The spreadsheet will have four columns. The first is the list of wavelengths, the
second is the extraterrestrial (AM0) irradiance, the third is the normal-incidence irradiance for AM1.5 and the third is the total (including scattered radiation) AM1.5
irradiance.
(a) Make a plot of the total irradiance as a function of wavelength. Be sure to
label the axes (including units) of the plot.
(b) From this spectra, calculate and plot the photon flux density (the number of
photons per area per time per energy) as a function of photon energy.
7

(c) Using numerical integration, calculate the total photon flux (number of photons per m2 per s) which have an energy above the bandgap of Si (Eg = 1.12
eV).
Solution:
There is a good explanation of this problem in the notes (Lecture 2, Slides 8-10).
However, here it is again.
(a) See Fig. 1.4

Figure 1.4: The AM1.5 spectrum


(b) The data you downloaded from the website is in the form of irradiance (I())
versus wavelength. We would like to find the photon flux density (n(E)) versus
energy. To calculate n(E), we will need to first calculate P (E), the irradiance
in terms of energy, instead of wavelength. From P (E) we can then get n(E)
for n(E) = P (E)/E. To get P (E) we recognize that
I()d = P (E)dE
We can then solve for P (E) as
P (E) = I()

d
dE

We know that wavelength and energy are related by E = hc/, so we can


write
8

hc
d
hc

= 2
E
dE
E

Then
P (E) = I()

hc
E2

and

n(E) =

I() Ehc2
hc
P (E)
=
= I() 3
E
E
E

where h and c are Plancks constant and the speed of light, respectively. To
plot the data, we need to convert the ordinate into energy units as well through
E = hc/. The converted plot is shown in Fig. 1.5.

Figure 1.5: The AM1.5 spectrum in terms of photon flux per energy with integrated
number of photons shown in blue and indicated by the shaded region of the curve
(c) You are trying to calculate the area under the n(E) curve to determine the
total photon flux over the different photon energies (or wavelengths) that
are above the bandgap energy of Si (1.12 eV). This can be accomplished by
integrating the n(E) curve over the appropriate limits. However, since our
data is discrete, we have to replace the integral with a sum and numerically
integrate the data. We need to be careful, though, because the n(E) data is
9

not regularly spaced in energy so we will need to compute E for each data
point. The number of photons can be found by

n(E)dE

Total photons =
1.12 eV

E
X

n(E)E(E) = 2.73 1021 photons

E=1.12 eV

6. (From the Midterm Exam in 2007) Despite conservation efforts, an on-campus


faculty home has a $1053 annual heating bill for natural gas.
(a) Assuming the price of gas (including taxes) is $0.0125/cubic foot, how much
energy was used in the year? Give your answer in both J/year and kWh/year.
(Note: 100 cubic feet of gas contain approximately one therm of heat energy.
There are 100,000 BTU in one therm, and there are about 1055 Joules in one
BTU.)
(b) What is the yearly averaged power (in kW) for heating?
(c) Assuming the gas heater is 80% efficient, and that an electric heater would be
100% efficient, how many square meters of 12% efficient solar panels would be
needed to satisfy the heating need? Assume the average solar flux density is
239 W/m2 .
Solution
(a)
$1053
1 ft3
1 thm 100, 000 BTU 1055 J

year $0.0125 100 ft3


thm
BTU
J
= 8.89 1010
year
Ws
h
kWh
= 8.89 1010

year 3600 s 1000 Wh


kWh
= 24687
year

Energy =

10

(b) The average power is given by:


24687 kWh 1 year 1 day

year
356 days 24 h
= 2.81 kW

Power =

(c) The area needed is given by:


2.81 kW 0.80 1000 W

239 W/m2 0.12 1 kW


= 78.6 m2

area =

7. The term horsepower was coined by the engineer James Watt (1736 to 1819) in
1782 while working on improving the performance of steam engines. This occurred
while using a mine pony to lift coal out of a coal mine. He conceived the idea of
defining the power exerted by these animals to accomplish this work. He found
that, on the average, a mine horse could pull (lift by means of a pulley) 22,000
foot-pounds per minute. Rather than call this pony power, he increased these
test results by 50 percent, and called it horsepower i.e. 33,000 foot-pounds of work
per minute.1 This works out to a power output of 745.7 Watts/horsepower.
If you have a solar cell with 15% efficiency and area of 3 m2 , how many tons (2000
pounds) of coal could you lift from a mine that is 1000 ft deep in one hour with an
incident radiation or 1.35 kW/m2 ? Assume that the solar cell is connected to an
electric motor that substitutes directly into the machinery where the mine ponies
would have been working and that the motor converts from electric energy to mechanical energy with 80% efficiency.
Solution:
The energy available to lift coal is the solar electricity times the motor efficiency.
By equating this to the gravitational potential energy we find:
mg =

A S t solar motor
h

where mg is the weight that can be lifted (mass times the acceleration due to
gravity), A is the solar panel area, S is the solar power density hitting the panels, t
1

from http://en.wikipedia.org/wiki/Horsepower

11

is the time, h is the height lifted, and solar and motor are the solar panel and motor
efficiencies. Putting in numbers we find:
3m2 1.35 kW/m2 60 min 0.15 0.80
1 hp
33000 ft lb 1 ton

1000ft
0.7457 kW
hp min
2000 lb
= 0.645 ton

mg =

12

Chapter 2
Semiconductor Physics
1. In class we saw that the electron occupancy of a given state with energy  is given
by the Fermi-Dirac distribution function:
f () =

1
exp[( )/kB T ] + 1

where is the electron chemical potential (known as the Fermi level), T is the
temperature and kB is Boltzmann constant, given by:
kB = 1.38 1023 J/K
= 8.62 105 eV/K
(a) Make a plot of this for temperatures T = 0.1 K, T = 300 K, and T = 2000 K.
For your plot, use an energy range from -3 eV to 3 eV, and take = 0.
(b) What is the value of f ()?
(c) Find the difference in energy between the points where f () = 1/4 and f () =
3/4. Your answer should be in terms of kB T .
(d) Describe the behavior of the Fermi-Dirac distribution at extremely high temperatures.
Solution
(a) See figure 2.1. Plotting this function is easy if you choose to use enough data
points in Excel or a program like Mathematica that can plot functions.
13

Figure 2.1: Fermi-Dirac Distribution Function for T = 0.1 K, T = 300 K, and T = 2000
K
(b) f() is the value of the Fermi-Dirac distribution function when the energy is
equal to the chemical potential.
f () =

1
1
1
=
=
exp[( )/kB T ] + 1
exp[0] + 1
2

(c) We begin by inverting the Fermi-Dirac distribution function to find:




1
 = kB T ln
1
f ()
So the energy difference  between the points where f () = 1/4 and f () =
3/4 is then given by:
 



1
1
 = kB T ln
1 ln
1
1/4
3/4
= kB T [ln(3) ln(1/3)]
= 2kB T ln(3)
= 2.197 kB T
14

(d) At high temperature kB T   so





1
exp
kB T
So f () 1/2.
2. The Ge band gap is EG = 0.66 eV, the density of states electron effective mass
is me = 0.55 m and the density of states hole effective mass is mh = 0.37 m.
The Si band gap is EG = 1.1 eV, the density of states electron effective mass is
me = 1.18 m and the density of states hole effective mass is mh = 0.81 m
(a) Plot the intrinsic carrier density as a function of temperature (from 300K 800K) for Ge and Si on the same graph. Why does Ge have a higher carrier
density than Si at comparable temperatures.
(b) The Si and Ge are then doped with 1013 atoms/cm3 and 1015 atoms/cm3 of
phosphorus respectively. Plot the carrier density (only electrons) as a function
of temperature (from 300K - 800K) for both on the same graph. Assume all
the dopants are ionized.
Note: You will have to use a log scale for the y-axis to fit the data for Si and Ge
on the same graph.
Solution
(a) The intrinsic carrier density is given by:

3/2


kB T
Eg
3/4
ni = 2
(mp mn ) exp
2~2
2kB T
Plugging in the numbers given for Eg , me , and mh , Plancks constant ~ =
1.055 1034 Js, Boltzmann constant kB = 1.38 1023 J/K = 8.62
105 eV/K, and varying temperature we can plot the curves shown in figure 2.2.
So we see that the smaller band gap for Ge results in a much higher intrinsic
carrier density than Si.
(b) For doped semiconductors, the carrier density is given by:

ND
+
n =
2
15

"

ND
2

#1/2

2
+

n2i

Figure 2.2: Intrinsic carrier concentration in Si and Ge


We can plot n as a function of temperature by plugging in the intrinsic concentration from the previous part of the question for different temperatures.
This is shown in figure 2.3.
3. A piece of silicon is doped with 81010 atoms/cm3 of Boron and 31010 atoms/cm3
of Phosphorus. Assume all the dopants are ionized and the temperature is 300 K.
(a) Is this material n-type or p-type?
(b) Calculate the concentrations of electrons and holes in the conduction and
valence bands, respectively.
(c) Find the Fermi level (with respect to the conduction band edge) of this material.
Solution
(a) The general neutrality condition is n + NA = p + ND , giving:
p n = NA ND
Since the concentration of boron (a group III acceptor) is greater than that
of phosphorous (a group V donor), NA > ND , so this material will be have
p > n and so will be p-type.
16

Figure 2.3: Carrier concentration in doped Si and Ge


(b) We modify the treatment in the notes which found the electron concentration in n-type material to find the hole concentration in this p-type material.
Solving the neutrality condition for n we find:
n = p + ND NA
Inserting this into np = n2i and solving for p we find:
NA ND
p=
+
2

s

NA ND
2

2
+ n2i

Inserting numbers we find:


p = 5.19 1010 cm3
The concentration of electrons can then be found from:
n2i
p
= 1.9 109 cm3

n =

17

(c) To find the Fermi level we use the expression derived in class:


( Ev )
p = Nv exp
kB T

Solving for we find:



= Ev + kB T ln

Nv
p

Using the result above and Nv = 1.83 1019 cm3 we find:


= Ev + 0.51 eV
This is a little below the middle of the gap.
4. (a) A 4mV voltage is applied across the block of silicon shown in Figure 2.4, with
length L = 8 cm and cross section area A = 6 cm2 . It generates a current of
2A in the circuit. Determine the number of charge carriers in the block. (The
mobility of Si is given by Si = 1400 cm2 /V s). Is this material doped?

V
I
L

Figure 2.4: Schematic of Si block and circuitry.


(b) You are given another rectangular block of the same type of silicon, a voltage control power supply (generates a specific voltage across two terminals),
and an ammeter (measures current moving through any circuit). Set up an
experiment to determine the dimensions of this new block without the use of
a ruler. You may need to use some of the information from part (a). (Hint:
You will have to take more than one measurement and then set up a system
of three equations and three unknowns.)
Solution
18

(a) Given the applied voltage and the current induced due to that voltage we can
determine the resistance of the block of material using
V =I R
The resistance of the block is related to the resistivity of the block and the
block dimensions by the formula
R=

L
A

where L is the length of the block in the direction of the current, A is the
cross-sectional area of the surface perpendicular to the length, and is the
resistivity of the silicon. We know that the number of charge carriers in the
material are related to the conductivity by
=ne
where is the conductivity, n is the concentration of charge carriers and
is the mobility of Si. Since the conductivity is inversely proportional to
resistivity,
= 1/
we can put this all together to get concentration of charge carriers in terms of
quantities that can be determined from what is given in the question
n=

n=

4 mV 6

cm2

LI
V Ae

8 cm 2 A
= 2.98 1018 carriers/cm3
19
2
1.6 10
J 1400 cm /V

(b) With the two pieces of equipment that are given to you the only experiment
you can really do is apply a voltage and measure the current generated in the
block. From this, we can calculate the resistance of the block (see above).
19

The key to this problem lies in the fact that resistance is dependent on the
dimensions of the block and resistivity is an inherent material property and
is constant (assuming constant temperature) no matter how the resistance
measurement is taken. Lets say we apply a voltage and generate a current in
the x direction. We can now calculate the resistance in the x direction and
then relate it to the resistivity and the dimensions of the block. (see figure
2.5)

Rx =

x
yz

Figure 2.5: Silicon block with unknown dimensions


where is the resistivity of the material (calculated in the main part of the
problem) and x,y, and z are the dimensions of the block in their respective
directions. Unfortunately, we now have one equation and three unknowns. To
solve for x,y, and z we will need two more equations that come from taking
the resistance measurement in the y and z directions.

Ry =
20

y
xz

Rx =

z
yx

Now we have three equations and three unknowns and can solve for x,y, and
z.
s

2
Rz Ry

2
Rz Rx

2
Rx Ry

x=

y=

z=

5. Semyon the Semiconductor Maker wants to make a silicon temperature sensor for
his oven. The sensor works by detecting the carrier concentration in a piece of
silicon and correlates this concentration to the temperature. Ideally, hed want a
piece of intrinsic silicon for this application, but intrinsic silicon is expensive and
hard to find (as Frank the TA found out the hard way). Therefore, he must make
do with doped silicon.
Semyon only bakes at temperatures above 250 F (he gets his recipes from an American website). What is the highest doping concentration that Semyon can tolerate
(and still have a reliable sensor for the temperature range)?
Solution:
In order to have a appreciable temperature dependence Semyon will need to have
a dopant level low enough so that his Si is intrinsic in the temperature range of
interest. So we need
ni (Ta ) ND,A
where Ta is the annealing temperature and ND,A is the doping level (either donor
21

or acceptor). Using the expression for ni in the notes we find:



ni = 2

kB T
2~2

3/2

4.67 1015


Eg
(me mh ) exp
2kB T


6499 K
3/2
T exp
cm3
T


3/4

where we have used:


me = 1.18m ,

mh = 0.81m ,

Eg = 1.12 eV

Inserting T = 250 F 394 K we find that the maximum doping level is:
ND,A 2.5 1012 cm3
6. Imagine that you engineered a solar cell where the composition varied with depth
(a likely consequence of heavy doping) and, as a result, the absorption coefficient
also varied with depth. Assume that the linear absorption coefficient is now a
function of depth as (z) = 0 z. With this new absorption coefficient, find an
expression for the photon flux of monochromatic light as a function of depth into
the semiconductor.
Solution: We must recognize how we arrived at our first expression for the light
intensity as a function of depth. We wrote out how the intensity changes across a
small distance element as
dI = I dz
Here we need to replace with (z) and continue as from before.
dI = (z)I dz = 0 zI dz
We can then integrate to get that
I (z) = I0 e

0 z 2
2

Where we have recognized the derivative through the reverse chain rule.
22

7. (From 2008 Midterm) The intrinsic temperature TI is the temperature at which the
intrinsic carrier concentration is equal to the dopant-induced carrier concentration
(the number of carriers introduced by the dopant).
(a) Given a donor dopant concentration of ND , find the intrinsic temperature in
terms of the doping concentration, effective densities of states and the band
gap.
(b) Evaluate this for the case of Si doped with a phosphorous concentration of
1016 cm3 Full credit will be given for assuming the effective densities of states
are constant and equal to their room temperature values, but if you wish you
can include their temperature dependence by iteration.
Solution:
(a) The intrinsic carrier concentration is given by:
n = (Nc Nv )1/2 E Eg /2kB T
where Nc and Nv are the effective density of states for the conduction and
valence band respectively. Setting this equal to the donor density we and
solving for temperature we find:
 
1
Nc Nv
Eg
ln
TI =
2kB
ND
This is not a closed form equation, since the effective densities of states depend
on temperature also.
(b) Inserting numbers using the 300 K effective densities of states we find:
 
1
1.12 eV
3.22 1019 1.83 1019
TI =
ln
2 8.62 105 eV/K
1016
= 833 K
Iterating the above equation a few times using the temperature dependence
of the effective densities of states we find:
TI = 714 K
8. (From 2013 Final Exam) We found that the concentration of electrons in the conduction band of a semiconductor is given by:
Z
n=
De ()fe () d
Ec

23

with the occupancy of electron states given by:


fe =

1
exp [( )/kB T ] + 1

and
1
D() = 2
2

2me
~2

3/2

( Ec )1/2

In class we found the result as:


3/2

me kB T
exp [ (Ec ) /kB T ]
n=2
2~2
However, to get this result we cheated. To get this result we made the approximation that the occupancy of electron states is given by:
fe () = exp [ ( ) /kB T ]
By examination we see that this approximation is valid only if:
Ec  kB T
Thus when the doping concentration is low, and the fermi level is far away from
the band edge, our approximation is ok. However, at high doping concentrations
it breaks down.
(a) What is the value of Ec that will give a 10% error in the occupancy of
electron states at the conduction band edge? Your answer should be a number
times kB T .
(b) For Si at 300 K, what doping concentration does this correspond to? For this
problem, use the regular (approximate) relationship between the doping concentration and the fermi level, and assume only donor doping, all the dopants
are ionized and that the doping concentration is large compared to the intrinsic
carrier concentration. Your answer should be in cm3 .
Solution:
(a) The error is given by:
fe (Ec ) fe (Ec )
e=
fe (Ec )
24

Inserting our expressions and simplifying we find:


exp [ (Ec ) /kB T ] = 0.1
or
Ec = ln(0.1)kB T = 2.3 kB T
(b) For Si the relationship between the doping concentration and the fermi level
is given by:
ND = Nc exp [ (Ec ) /kB T ]
So the doping concentration for this level of error is given by:
ND = 0.1 Nc = 3.22 1018 cm3

25

26

Chapter 3
Photovoltaic Devices

1. Monochromatic light with photon flux n0 photons per unit area per second is normally incident on a semiconductor surface. A fraction R is reflected.
(a) Find an expression for n(z), the photon flux as a function of the depth z
into the semiconductor. Assume that the this light has a linear absorption
coefficient in this material.
An important quantity in modeling solar cell performance is the generation rate
g(z), which is the rate per volume at which light-induced carriers (electron-hole
pairs) are generated. This quantity is defined by noting that g(z) dz is the number
of carriers per area per time generated in the region between z and z + dz.
(b) What are the units on g(z)?
(c) Assuming that each photon absorbed creates one electron-hole pair, and using
the parameters mentioned above, calculate an expression for the generation
rate g(z) of these pairs as a function of depth in the semiconductor.
Solution
(a) Since the light is monochromatic, we dont have to worry about variations in
the absorption with energy. The photon flux goes simply as the intensity as
we have seen from class. To account for the reflected light at the surface we
just change n0 to n0 (1 R) and we get, for the photon flux as a function of
27

depth
n(z) = n0 (1 R)ez
(b) The units on g(z) are number per time per volume.
(c) Now the generation rate is just the amount by which the photon flux decreases
with depth, which is the negative derivative of the photon flux as a function
of depth.
g(z) =

dn
= n0 (1 R)ez
dz

2. Using the website http://www.ee.byu.edu/cleanroom/OpticalCalc.phtml to find


the absorption coefficients for Si and GaAs at each wavelength, compare the depths
where the photon flux is reduced to %10 of the value just as the light enters the
material for 700 nm and 400 nm light.
Note: The depth you will calculate is the minimum thickness your solar cell needs
to be to absorb a significant fraction of the light.
Solution
From the website listed in the problem set, we get the data shown below.

400 nm
700 nm

Si

GaAs

121579 cm1
2345.5 cm1

674185 cm1
27424 cm1

We want the distance where the photon flux is 10% of that at the surface, so we
use the formula from class.
I
ln(0.1)
= ez = 0.1 z =
I0

Plugging in the values of the absorption coefficients from above, we get the values
below.
3. A p-n junction is created by doping the right side of a piece of silicon with 1014 atoms/cm3
of phosphorus and the left side with 1018 atoms/cm3 of Boron. Enough time is given
for the junction to come to equilibrium (ie. drift current = diffusion current).
28

Si
400 nm 0.19 m
700 nm 9.8 m

GaAs
0.034 m
0.84 m

(a) Draw a band diagram of the p-n junction, mark Ec , Ev , f,intrinsic , and f .
This plot does not have to be to scale, but the relative positions of the energy
levels must be correct.
(b) Plot the charge density and electron concentration (in C/cm3 ) as a function
of position in the silicon piece. This plot does not have to be to scale, but the
relative magnitudes of the charge density (on each side of the junction) and
electron concentration must be correct.
(c) Determine xn , xp (length of depletion region on the n side and the p side),
and the total depletion width W .
(d) Plot the electric field as a function of position (take into account the differences
in xn and xp ).
(e) Plot the voltage as a function of position (take into account the differences in
xn and xp ).
Notes:
Plots may be hand drawn for this question.
Assume x=0 is at the center of the p-n junction
Solution
(a) Figure 3.1 shows the band diagram of the p-n junction that is formed. Notice
that the distance between the Fermi Level and the conduction band on the n
side is larger than the distance between the Fermi level and the valence band
on the p side.
(b) Figure 3.2 shows the space charge density and the electron concentration in
the p-n junction. While the electron concentration is zero in the depletion
width, it is non-zero on the p-side outside the depletion width because the
electrons are the minority carrier on that side.
(c) Xn , Xp and W are all only dependent on the doping concentrations and the
29

Figure 3.1: Energy Band Diagram for P-N Junction


built in voltage of the junction.
s
Xp =
s
Xn =

2s Vbi ND
eNA (NA + ND )
2s Vbi NA
eND (NA + ND )

W = Xp + Xn
Vbi is dependent on the doping concentrations, the intrinsic carrier concentration of Si and the temperature. If we assume room temperature.

Vbi =

kB T NA ND
1.38 1023 295 1018 1014
ln
=
ln
= 0.703 V
e
n2i
1.6 1019
(1010 )2

Plugging into the equations for Xp and Xn


s
xp =

2 1.04 1012 0.703 1014


= 3.022 108 cm = 3.022
A
1.6 1019 1018 (1018 + 1014 )
30

Figure 3.2: Space Charge Distribution and Electron Concentration

s
xn =

2 1.04 1012 0.703 1018


= .0003052 cm = 3.052 m
1.6 1019 1014 (1018 + 1014 )
W = xn + xp = 3.052 m

(d) Figure 3.3 shows the E-field and Voltage in the PN junction. The Electric
field is related the charge concentration by
Z b
E=
(x)dx
a

It is related to the potential by


Z
V =

E(x)dx
a

31

(e) See part d.


4. In a previous homework problem, you were asked to calculate and plot the photon
flux as a function of photon energy for AM1.5, and then calculate the total photon
flux above the band gap of Si. We assume that each photon above the band gap is
able to produce an electron, and the voltage at which they are extracted is equal
to the band gap (Vbg ). The power per area produced will then be JL Vbg , where JL
is the light current per area, which in this case is equal to electron charge times the
flux of photons with energy above the band gap energy (eVbg ).
(a) Plot the power per area as a function of band gap energy eVbg and determine
the optimum band gap for solar cell applications (band gap that produces the
most power).
(b) What solar cell efficiency does this correspond to?
Solution
(a) In Problem #4 of Homework #1 you calculated the photon flux density from
the Irradiance by the following method:
I()d = P (E)dE
We can then solve for P (E) as
P (E) = I()

d
dE

We know that wavelength and energy are related by E = hc/, so we can


write
=

hc
d
hc

= 2
E
dE
E

Then
P (E) = I()
and
32

hc
E2

I() Ehc2
P (E)
hc
n(E) =
=
= I() 3
E
E
E
where h and c are Plancks constant and the speed of light, respectively. To
plot the data, we need to convert the ordinate into energy units as well through
E = hc/. The converted plot is shown in figure 3.4.
You were also asked to determine the total photon flux above the bandgap of
Si. This can be accomplished by integrating the n(E) curve over the appropriate limits. However, since our data is discrete, we have to replace the integral
with a sum and numerically integrate the data. We need to be careful, though,
because the n(E) data is not regularly spaced in energy so we will need to
compute E for each data point. The number of photons can be found by:

n(E)dE

Total photon flux =


Eg eV

E
X

n(E)E(E)

E=Eg eV

The total photon flux above the bandgap of a material is the photon flux that
is absorbed and generates electrons. Thus, if we use this method to calculate
the photon flux above each bandgap energy we would get the green dashed
curve in figure 3.4. This curve has a maximum at zero and decreases with
energy. This makes sense because if we make the bandgap infinitely small,
all the incident photons will be absorbed by the material. As the bandgap
increases, the photon flux that is absorbed decreases as well because there is a
smaller number of photons with energies large enough to excite electrons over
the bandgap.
If we assume that each photon above the bandgap generates an electron in
the material that contributes to the current, we can calculate the total current
generated by the light as a function of bandgap:
charge
=e
Current Density(Eg ) =
time area
33

n(E)dE
Eg

The question however, asks for power density as a function of band gap energy.
To convert current density to power density we need to know the voltage that
electrons are extracted it. In a real solar cell, this voltage is the V oc calculated
from an IV curve. We assume that this voltage is equal to the band gap and
solve:

Power Density(Eg ) = Voltage Current Density(Eg ) = e Eg

n(E)dE
Eg

This curve is the blue curve plotted in figure 3.4. The maximum occurs at a
band gap of 1.11ev and 441 W/m2 .
5. (From the Final Exam in 2007) In a linearly graded PN junction, the space charge
density can be varied as a function of position by varying the doping concentration.
The charge density is given by (x) = x where is some constant. Figure 3.5
shows a plot of this function. X0 is the magnitude of the depletion width on each
side of the junction.
(a) Mark the p and n side of the junction.
(b) Derive an expression for the electric field as a function of position and plot it.
(c) Determine the maximum electric field magnitude in terms of X0 and .
(d) Derive an expression for the voltage as a function of position and plot it.
(e) Determine Vbi in terms of X0 and .
Notes:
Assume x=0 is at the center of the p-n junction.
It is convention to integrate any position functions from the left to right.
Solution:
(a) Since the holes diffusing out of the p-type region will leave behind a negative
charge, the p-type region is to the left.
34

(b) Using Poissons equation we find:


Z

(x)
dx
s
Z x
x
=
dx
x0 s
2
=
(x x20 )
2s

E =

(c) The maximum electric field magnitude occurs at x = 0 and has value:
Emax =

x20
2s

This is the magnitude. The actual field value at that point is negative.
(d) The voltage is found from by integrating again:
Z x
V =
E(x) dx
x0
Z
x 2
(x x20 ) dx
=
2s x0


2x30 x3
2
=
+ xx0
2s
3
(e) The total bias voltage is found by evaluating the voltage at x = x0 . This
gives:
2x30
Vbi =
3s
6. A p-n junction is created by doping the left side of a piece of silicon with 1014 atoms/cm3
of phosphorus and the right side with 1018 atoms/cm3 of Boron. Enough time is
given for the junction to come to equilibrium (ie. drift current = diffusion current).
A voltage is then applied to Reverse Bias the junction.
(a) Draw two band diagrams, one of the standard p-n junction and the other of
the junction after the voltage is applied.
Mark Ec , Ev , intrinsic , f , Vbi , Va , n , p , xn , xp , and W on both graphs. You
do not have to draw the graph with exact values but the relative values of
these parameters should be drawn accurately and clearly.
35

(b) Determine the direction and relative sizes of the electron drift and diffusion
currents as well as the hole drift and diffusion currents for both the unbiased
and biased junction.
(c) Draw the charge density for both conditions on the same graph to show the
difference between the biased and unbiased case. Mark Na , Nd , xn , xp .
(d) Draw the E-field for both conditions on the same graph to show the difference
between the biased and unbiased case.
(e) Draw the voltage for both conditions on the same graph to show the difference
between the biased and unbiased case. Mark xn and xp .
(f) Finally, we shine some light on the unbiased junction in an attempt to use
it as a solar cell. Explain what happens in the junction to produce current.
Concentrate on the changes in the drift and diffusion currents and why they
occur. You may use words, figures and equations to explain yourself.
Notes:
Make sure all your graphs are oriented such that the phosphorus is on the left
and the boron is on the right.
Assume x=0 is at the center of the p-n junction.
It is convention to integrate any position functions from the left to right.
Solution:
(a) Figure 3.8 and figure 3.9 show the band diagrams for an unbiased and biased
junction respectively.
(b) Figure 3.10 shows the drift and diffusion currents for electrons and holes. The
size of the arrow indicates the magnitude of the current.
(c) Figure 3.11 shows the charge density, electric field and voltage of the biased
and unbiased junctions.
(d) See part c.
(e) See part c.
(f) When you shine light on the junction, photons from the light are absorbed
by interacting with electrons in the semiconductor. In the absorption process, the energy from the absorbed photons is transferred to the electrons.
36

Electrons in the valence band can absorb photons with energy sufficient to
promote them to the conduction band, leaving behind a hole in the valence
band. So absorption of light results in electron-hole pairs in the semiconductor. Electron-hole pairs produced in p-type region of the p-n junction, produce
excess minority electrons. These minority electrons can lower their energy by
moving down the electron energy profile into the n-type region. This is an
increase in the drift current. Similarly, electron-hole pairs produced in the
n-type region produce excess minority holes, which move toward lower hole
energy in the p-type region, also increasing the drift current. This movement
of minority electrons from the p-type to the n-type region or minority holes
from the n-type to the p-type region through the junction separates the electron and hole in the electron hole pair. The resulting increase in drift current
breaks the balance between the drift and diffusion current in the junction.
Power can be extracted from this current as work can be done as the minority
carriers are driven through the junction by the built-in voltage.
7. (From 2008 Midterm) We consider a Si p+ n junction with p+ region doping of
NA = 1018 cm3 and n region doping of ND = 1016 cm3 at a temperature of
T = 350 K. Assume all the dopants are ionized and take the top of the valence
band to be at E = 0.
(a) Find the fermi level in the p+ region.
(b) Find the fermi level in the n region.
(c) What is the built-in voltage for this junction?
Solution:
(a) The fermi level can be found from our expression for the hole concentration:


( Ev )
p = Nv exp
kB T
Solving for and inserting numbers we find:

p+ = Ev kB T ln

p
Nv

= 0 + 0.088 eV
= 0.088 eV
where in the last steps we used the parameters for Si.
37

(b) We can find the fermi level in the n region using the expression for electron
concentration:


(Ec )
n = Nc exp
kB T
Solving for and inserting numbers we find:
 
n
n = Ec + kB T ln
Nc
= 1.12 eV 0.24 eV
= 0.88 eV
where in the last step we have again used the parameters for Si.
(c) The built in voltage will be the difference between the chemical potentials in
the two regions:
bi = n p+

 
p
n
Ev kB T ln
Ec + kB T ln
N
Nv
 c 
np
Eg + kB T ln
Nc Nv
1.12 eV 0.33 eV
0.79 eV


=
=
=
=

8. (From 2011 Midterm) A heterojunction solar cell is two materials with different
band gaps. For this problem we take material A to have a band gap of 1 eV and
material B to have a band gap of 1.5 eV. We assume that the the temperature
is 300 K, and assume that the effective density of states is the same for both the
valence and conduction band for both materials and is given by its free electron
value:
3/2

mkB T
= 2.51 1019 cm3
Nc = Nv Nc,v = 2
2~2
where m is the free electron mass.
(a) What are the intrinsic (un-doped) electron and hole carrier densities for each
material?
(b) What is the position of the fermi level in each isolated material when they are
both intrinsic (un-doped). Measure the position of the fermi level relative to
the top of the valence band.
38

We now dope material A with n-type dopants to a density of 1 1012 cm3 and we
dope material B with a p-type dopants to a density of 1 1017 cm3 . We assume
that all the dopants are ionized.
(c) What is the carrier density in each isolated material? Find both n and p for
both materials.
(d) What is the position of the fermi level in isolated material A and material B.
Again, measure the position of the fermi level relative to the top of the valence
band.
We now bring the doped A and B together to form a p-n junction. Eventually the
diffusion current is countered by the drift current of the built in potential, as in a
p-n junction formed from the same materials.
(e) Sketch the electron energy level diagram for material A and B before and after
forming the p-n junction. Show on you sketch the fermi level, the intrinsic
fermi levels, as well as the conduction and valence band edges.
Solution:
(a) The intrinsic electron and hole carrier density is given by:

1.0 1011 cm3 material A
Eg /2kB T
ni = pi = Nc,v e
=
6.3 106 cm3 material B
(b) Since the effective densities of states are the same for both the conduction and
valence band, the fermi level will be right in the middle of the gap for both
materials So for intrinsic material A, the fermi level will be 0.5 eV above the
top of the valence band and for intrinsic material B, the fermi level will be
0.75 eV above the top of the valence band.
(c) Turning first to material A we first calculate the electron density:
s
2 
2
ND
ND
(A)
(A)
+
+ ni
n
=
2
2
= 1.01 1012 cm3
We find the hole concentration by:

2
(A)
ni
p(A) =
= 9.90 109 cm3
n(A)
39

Similarly for material B we find:


p

(B)

NA
=
+
2

s

NA
2

2

2

(B)
+ ni

= 1.00 1017 cm3


We find the electron concentration by:
2

(B)
ni
n(B) =
= 3.98 104 cm3
p(B)
Wow! This is really small.
(d) The fermi level position can be found from the equation for the carrier density.
For electrons in material A we have:
n = Nc,v e(Ec )/kB T
Rearranging we find:
Ev = EG (Ec )


n
= EG + kB T ln
Nc,v

0.56 eV material A
=
0.14 eV material B

40

41
Figure 3.3: E-field and Voltage
in the PN Junction

400

x10

21

300

200

100

Figure 3.4: Photon flux density and power density vs. band gap energy

Figure 3.5: Charge density for a linearly graded PN junction

42

Figure 3.6: Electric field versus position for graded composition p-n junction.

Figure 3.7: Potential versus position for graded composition p-n junction.

43

Figure 3.8: Band Diagram for Unbiased Junction

Figure 3.9: Band Diagram for Biased Junction

44

Figure 3.10: Drift and Diffusion Currents

45

Figure 3.11: Drift and Diffusion Currents

46

Chapter 4
Solar Cells
1. Download the file HW4IVdata.txt from the coursework website. The file contains
actual current vs voltage data taken for an organic solar cell.
(a) Make a plot of current vs. voltage.
(b) Make a plot of power vs. voltage and determine the maximum power produced
by the cell.
(c) What is the optimum load resistance to operate this cell at?
(d) Calculate the fill factor.
Solution
(a) See figure 4.1.
(b) See figure 4.1.
(c) The voltage which corresponds to the maximum power is Vmax = 247 mV and
the corresponding current is Imax = 0.259 mA. Hence the resistance which
would lead to optimum power would be:
Rmax =

Vmax
= 954
Imax

(d) From the data the open circuit voltage and short circuit current are given by:
Voc = 384 mV
47

and

Isc = 0.372 mA

The fill factor is then:


Imax Vmax
Isc Voc
= 0.45

FF =

Figure 4.1: Top: Current-voltage curve for organic solar cell, showing shaded region
corresponding the the maximum power condition. Bottom: Power (P = IV ) as a function
of voltage for organic solar cell.
2. A silicon diode has doping in the p and n regions of NA and ND respectively. The
minority electrons in the p region have diffusion length Ln and diffusivity Dn , and
the minority holes in the n region have diffusion length Lp and diffusivity Dp . The
values for these are given in the table below.
3

ND (cm )
2 1016

n Region
Lp (cm)
11.4 103

Dp (cm /s)
13

Also take T = 300 K and ni = 1010 cm3


48

NA (cm )
2 1017

p Region
Ln (cm) Dn (cm2 /s)
4.1 102
34

(a) Find Js .
(b) Find the current density when this diode is forward biased with 0.66 V.
(c) Make a plot of the current density as a function of voltage for this diode.
Now we illuminate this junction with the spectra n(E) shown below.
(d) Assuming each photon above the band gap produces a carrier, what is the
light current density (JL ) through the cell?
(e) What is the total current density through the cell when it is biased with a
voltage of 0.66 V?
(f) Make a plot of the current density as a function of voltage for this cell when
it is illuminated.
(g) What is the maximum power density (W/cm2 ) for this cell?
(h) What is the efficiency for this cell?
(i) What is the open circuit voltage Voc for this cell under illumination?

4
AM1.5
5960 K Spectra

x10

1.0

1
0
0

21

1.5

x10

21

2.0

0.5

0.0

Figure 4.2: Solar photon spectra. Dotted line shows the total number of photons with
energy above the Si band gap as a function of energy. (The vertical axis for the dotted
line is on the right.)
49

0.12
) mc/A(

0.08
0.04

0.00
-0.04
-1.0

-0.5

0.0

0.5

1.0

Voltage (V)

Figure 4.3: Current density versus voltage for the p-n diode described in the problem.
Solution
(a) We can find Js by inserting values into the equation:


Dn
Dp
2
+
Js = eni
Lp ND Ln NA
= 9.8 1013 A/cm2
(b) The current density at a given voltage is given by:

J = Js eV /Vth 1
where Vth = kB T /e, which, at T = 300 K, has the value Vth = 0.0259 eV.
Taking V = 0.66 eV, we find:
J = 0.11 A/cm2
(c) The plot is shown below:
(d) Since each photon with energy above the band gap produces a carrier in the
semiconductor, the light current density is just the electron charge times the
50

number density of the photons with energy above the band gap. From the
figure we find
Z
JL = e
n(E) dE
EG



1
19
21
= 1.602 10
C 2.5 10
m2 s
= 4.0 102 A/cm2
(e) The current density for a solar cell is given by:

J = JL Js eV /Vth 1
Using the results from above for V = 0.66 V we find:
J = 4.0 102 A/cm2 0.11 A/cm2 = 0.07 A/cm2
so in this case, the current is running backward through the cell since JF > JL
(f) The current density versus voltage plot is shown below.
(g) The power density P/A = JV is shown plotted in figure 4.5. From this graph
we can see that the maximum power density is 21 mW/cm2 . We could also
have found the maximum power voltage Vmax by solving the equation
Vth eVoc /Vth = (Vmax + Vth ) eVmax /Vth
and found the maximum power by multiplying Vmax by the corresponding
current density. This gives



P
Vmax /Vth
=
V
J

J
e

1
max
L
s
A max

= (0.55 V) 38 mA/cm2
= 21 mW/cm2
(h) The efficiency for this cell can be found by dividing the maximum power
found above by the total input solar power. The total input solar power can
be found by integrating the AM1.5 spectra shown above. When I do so I get
1000 W/m2 = 10.0 mW/cm2 . Hence the efficiency is
21 mW/cm2
100 mW/cm2
= 21 %

51

0.04

) mc/A( J

0.00
-0.04
-0.08
-0.12
-1.0

-0.5

0.0

0.5

1.0

Voltage (V)

Figure 4.4: Current density versus voltage for the illuminated p-n diode solar cell described in the problem.
2

) mc/Wm( ytisneD rewoP

25
20
15
10
5
0
-1.0

-0.5

0.0

0.5

1.0

Voltage (V)

Figure 4.5: Current density versus voltage for the p-n diode described in the problem.
52

(i) The open circuit voltage Voc can be found from the plot in figure 4.4 or through
the expression from the notes


JL
Voc = Vth ln 1 +
Js
= 0.63 V
3. (From the Midterm Exam in 2007) An ideal (Rsh = , Rs = 0) Si p-n junction
solar cell with the following properties:
NA = 5 1018 cm3
Dn = 25 cm2 /s
n = 5 107 s

ND = 1016 cm3
Dp = 10 cm2 /s
p0 = 107 s

is exposed to the standard AM1.5 solar spectrum with an intensity of 900 W/m2
corresponding to a solar flux of photons with energy above the bandgap of 2.5
1017 cm2 s1 . The temperature of the device is held at 300 K.
(a) Find the light-induced current density JL Assume that a carrier is excited for
each incident photon with energy greater than the bandgap.
(b) Find the minority carrier diffusion lengths Ln and Lp .
(c) Find the reverse saturation current density Js .
(d) Find the open circuit voltage Voc .
(e) Again assuming ideal diode behavior, find the efficiency of this solar cell.
(f) What is the cell efficiency if we increase the solar flux by a factor of 100?
Solution:
(a) Assuming that each photon with energy above the band gap creates one carrier, the light-induced carrier density is just the the carrier charge times the
incident photon density:
Z
JL = e
n(Eph ) dEph
Eg

= 1.602 1019 C 2.5 1017 cm2 s1


A
= 0.040
cm2
53

(b) The minority carrier diffusion lengths Ln and Lp are given by:
p
1/2
Ln = Dn n = 25 cm2 /s 5 107 s
= 3.5 103 cm
and
Lp =

p
1/2
Dp p = 10 cm2 /s 107 s
= 1.0 103 cm

(c) The reverse saturation current density is given by:


eDn np0 eDp pn0
+
Ln
Lp


Dn
Dp
2
= eni
+
Ln NA Lp ND

Js =

where we have used n2i = np and set n = ND in the n-region and p = NA in


the p-region. Inserting numbers we find:
Js = 1.602 1019 C (1010 cm3 )2


10 cm2 /s
25 cm2 /s
+
3.5 103 cm 5 1018 cm3 1.0 103 cm 1016 cm3
= 1.604 1011 A/cm2
(d) Since for the ideal diode Jsc = JL , the open circuit voltage is given by:


kB T
JL
Voc =
ln 1 +
e
J
 s

0.040
= 0.026 V ln 1 +
1.604 1011
= 0.559 V
(e) From the attached graph for an ideal diode solar cell fill factor, we find that
for Voc = 559 mV the fill factor is given by F F = 0.818. Hence the efficiency
of this cell is given by:
Voc Jsc F F
Ps
0.559 V 0.040 A/cm2 0.818
=
900 W/m2
= 20%

54

(f) Increasing the solar flux by a factor of 100 increases JL by a factor of 100.
Thus the open circuit voltage becomes:


kB T
JL
Voc =
ln 1 +
e
J

 s
4.0
= 0.026 V ln 1 +
1.604 1011
= 0.678 V
From the attached graph we now find F F = 0.84 and thus the efficiency is:
Voc Jsc F F
Ps
0.678 V 4.0 A/cm2 0.84
=
90000 W/m2
= 25%

4. (From the Midterm Exam in 2007) As mentioned in class, advanced solar cells
have a back contact field associated with a p+ layer that is below the p layer of
a solar cell. To examine the effect of this layer we consider the simple case of a
p+ p structure as shown schematically below, where the Si p+ layer has a doping of
(p+ )
(p)
NA = 1018 cm3 and the Si p layer has a doping of NA = 1016 cm3
(a) Calculate the position (in eV) of the fermi level in an isolated p+ layer and
an isolated p layer with the above dopings. (Assume that the position of the
valence band edge is Ev = 0, and that the temperature is 300K.)
(b) Calculate the potential difference (in volts) across the junction between these
two layers.
(c) Sketch the fermi level, valence band edge and conduction band edge for this
p+ p structure.
Solution:
(a) The relationship between hole concentration, fermi level and valence band
edge is:
p = Nv exp[( Ev )/kB T ]
we can find:
= Ev + kB T ln
55

Nv
p

0.90

0.85

rotcaF lliF

0.80

0.75

0.70

0.65

0.60
200

300

400

500
V

oc

600

700

(mV)

Figure 4.6: Ideal diode fill factor as a function of open circuit voltage.

Figure 4.7: Schematic of p+ p structure.

56

800

Using Nv = 1.83 1019 cm3 from the notes, and p = NA for each layer we
find:


1.83 1019
(p+ )

= 0.026 eV ln
= 0.0756 eV
1018
and

(p)


= 0.026 eV ln

1.83 1019
1016


= 0.195 eV

(b) The difference in potential across the layer will be:


=

(p

+)

(p)
= 0.120 V
e

5. Figure 4.8 shows the IV curve and max power for this PN junction after it is made
into a Solar Cell.
(a) What would be the optimal load resistance to operate this cell at?
(b) If I change the top contacts of the cell to a higher conductivity material how
would that change the IV curve? Draw the new curve on the graph.
(c) Calculate the Fill Factor and efficiency. (assume that the light incident on the
sample has a power of 5 Watts)

Figure 4.8: IV Curve and Max Power for Solar Cell


Solution
57

(a) We know that the power coming from the cell is dependent on the current and
voltage that cell is operating at. The only way that we have of controlling
the current and voltage coming from the cell is by adjusting either the light
incident on it or the load resistor. Since in the operation of a real solar cell
we cannot adjust the light we are only left with the load resistor. The optimal
load resistance will allow us to extract the maximum power from the cell.
From the graph we can see that this point occurs at 195mV and 2.75mA. The
load resistance can then be found by

R=

195 mV
V
=
= 70.9
I
2.75 mA

(b) Changing the top contacts to a higher conductivity material lower the series
resistance of the cell. This increases the IV curve and the max power point.
See figure 4.9.

Figure 4.9: IV Curve and Max Power for Solar Cell

58

(c) The fill factor is given by


FF =

Imax Vmax
195 mV 2.75 mA
=
= 0.41
Isc Vsc
305 mV 4.3 mA

Efficiency is given by
=

Isc Voc F F
305 mV 4.3 mA 0.41
=
= 0.00011 = 0.011%
Ps
5W

6. (From 2008 Midterm) Due to the high cost of high-efficiency solar cells, many are
used with optical concentrators to increase the amount of light power per solar cell
area. The sunlight intensity is often measured in units of suns, so that the irradiance
for Ns suns would be Ns times the irradiance of an AM1.5 solar spectrum. We
wish to see how this increase in light changes the operation of the solar cell. For
simplicity we use an ideal solar cell where we can ignore the effects of the series and
shunt resistances, and the fill factor as a function of open circuit voltage is given
in figure 4.10. We also take
Js = 1012 A/cm2
and that an illumination of one sun (Ns = 1) with a total power density of
900 W/m2 produces a light current density of:
JL(1) = 4 102 A/cm2
Further we assume that the light current scales with the number of suns (JL(Ns ) =
Ns JL(1) ) and that the temperature of the cell is 25 C independent of the amount
of light.
(a) What is the open circuit voltage Voc for Ns = 1 and Ns = 500?
(b) What is the efficiency of the solar cell under these two conditions of solar
irradiance?
(c) What is the power per area for the solar cell under these two conditions of
solar irradiance?
Solution
(a) The open circuit voltage is given by:


IL
kB T
ln 1 +
Voc =
e
Is
59

Figure 4.10: Fill factor as a function of open circuit voltage for Si solar cells at room
temperature.

60

Inserting the numbers given above we find:


Voc (Ns = 1) = 0.63 V

and

Voc (Ns = 500) = 0.79 V

(b) The efficiency is given by


=

Pc
F F Voc Jsc
=
Ps
Ps

where Pc and Ps are the output power density of the cell and the input power
density of the sun respectively, F F is the fill factor, and Jsc is the short circuit
current density, which is equal to the light current density. Using the graph
in figure 4.10 we find the following:
Ps
Ns
1 0.090 W/cm2
500
45 W/cm2

Jsc
2

4 10 A/cm
20 A/cm2

Voc
630 mV
790 mV

FF

Pc
0.83 23% 0.021 W/cm2
0.86 30% 13.6 W/cm2

(c) The power per area is given by:


Pc = F F Voc Jsc
Using the numbers above we find the numbers in the table for part (b). We
note that we produce 650 times more power per area using only 500 times as
much light energy input.
7. (From 2009 Midterm) One approach for improving the efficiency of a thin film
solar cell is to pattern the junction as shown in Figure 4.11 so that photo-excited
carriers are always within a diffusion length to the junction, while still allowing for
a long optical penetration depth so all the sunlight is absorbed. One disadvantage
of this geometry is that the junction area is significantly increased, leading to an
increased dark current. For this problem we take the junction area to be AJ while
the area exposed to the sunlight is AL . Since we have a different solar light area
and junction area, we now re-examine the formalism for the cell current voltage
behavior. Assume ideal diode behavior.
(a) Write down the equation for the solar cell current I as a function of the reverse
saturation current density Js , the light-induced current density JL , the load
voltage V , temperature T , junction area AJ and sunlight exposure area AL .
(b) Find the equation for the open circuit voltage in terms of the reverse saturation current density Js , the light-induced current density JL , temperature T ,
junction area AJ and sunlight exposure area AL .
61

Figure 4.11: Schematic of structured solar cell with interdigitated n and p regions giving
a long light absorption pathway and a short minority carrier distance to the junction.
(c) Taking T = 300 K, JL = 0.04 A/cm2 , and Js = 1012 A/cm2 . Find the open
circuit voltage for the cases AJ /AL = 1 and AJ /AL = 10.
(d) If we assume our cell has thickness 3/, where is the optical absorption
coefficient, which corresponds to absorption of 95% of the light. We take
our pattern feature width to be 2Lm , where Lm is the minority carrier diffusion
distance. This gives a junction area of:1


3
AJ = AL 1 +
2Lm
Taking = 3 106 m1 and Lm = 100 nm, calculate the ratio AJ /AL .
Solution
(a) The solar cell I V curve is given by:




eV
I = AL JL AJ Js exp
1
kB T
(b) To find Voc we set the current equal to zero and solve for V . This gives:


AL JL
kB T
Voc =
ln 1 +
e
AJ Js
(c) Inserting numbers we find for AJ /AL = 1, Voc = 0.63 V and for AJ /AL = 10,
Voc = 0.57 V.
1

Dont spend time deriving this.

62

(d) Using the given expression we find:


AJ
3m
= 1+
AL
3 106 2 100 109 m
= 6
8. (From 2009 Midterm) As we have seen in one of the example problems, increasing
the light intensity can result in a saving of cell area and an increase in Voc and
hence efficiency. However the increased light can also heat up the cell, resulting in
a decrease in Voc and efficiency. In this problem address this issue.
Assume that the p-n junction being used has the following properties:
NA
cm3
1018

ND
cm3
1016

Dn
Dp
2
cm /s cm2 /s
30
15

Ln Lp E g me
mh
m m eV m
m
39 12 1.1 1.18 0.81

Where NA and ND are the acceptor and donor doping densities, Dp and Dn are the
hole and electron diffusivities in the n and p regions respectively, Lp and Ln are
the hole and electron diffusion length in the n and p regions respectively, Eg is the
band gap, mh and me are the hole and electron density of states effective masses,
and m is the electron mass. We also assume that the solar cell behaves as an ideal
diode with a fill factor of F F = 0.80.
For this problem we illuminate our solar cell with 500 suns. This gives a light
current density of
JL = 500 0.04 A/cm2 = 20 A/cm2
and a solar power density of
Ps = 500 900 W/m2 = 4.5 105 W/m2
We assume that this heats our solar cell to a temperature of T = 400 K.
(a) Find the intrinsic carrier density (ni ) for this cell under these conditions.
(b) Determine the reverse saturation current density (Js ) for this junction under
these conditions.
(c) Determine the open circuit voltage (Voc ) for this junction under these conditions.
(d) Calculate the efficiency of the cell under these conditions.
63

Hint: To save you time you might want to use the following shortcut. Given the
parameters for this problem the effective density of states for the conduction and
valence bands are given by:

3/2

3/2
T
T
19
3
Nc =
3.22 10 cm
and Nv =
1.83 1019 cm3
300 K
300 K
Solution:
(a) The intrinsic density is given by:
p
ni = Nv Nc eEg /2kB T
Using the hint and plugging in numbers we find:
ni = 4.4 1012 cm3
(b) The reverse saturation current is given by:


Dp
Dn
2
Js = eni
+
ND Lp NA Ln
Plugging in the numbers given for this cell we find:
Js = 3.8 106 A/cm2
(c) The open circuit voltage is given by:
Voc



JL
kB T
ln 1 +
=
e
Js

Plugging in numbers we find:


Voc = 0.53 V
(d) The efficiency is given by:
Voc JL
Ps
Once again plugging in numbers we find
= FF

= 19%
64

For comparison, under one sun we found:


ni = 1.41010 cm3 , Js = 3.91011 A/cm2 , Voc = 0.54 V and = 19%
So the increase in efficiency from the increased light current is almost exactly
canceled out by the decrease in efficiency due to the temperature increase.
Still, we make a lot more solar power per area of solar cell.
9. (From 2010 Midterm) Thin-film silicon solar cells typically use a p-i-n structure,
rather than a p-n structure. A p-i-n junction is similar to a p-n junction, except
that a layer of intrinsic material is sandwiched in between the p and n layers. He we
consider a p-i-n solar cell formed from p-doped, intrinsic, and n-doped Si. Assume
that the n-type silicon is doped with ND = 1018 cm3 donors, and the p-type is
doped with NA = 1015 cm3 acceptors. We consider a temperature of 300 K and
assume all the dopants are ionized.

Figure 4.12: Schematic of p-i-n solar cell.


(a) Start with separate pieces of n-type, intrinsic, and p-type Si. Draw the band
structures for each, labeling their Fermi levels.
(b) For the n and p materials, calculate the position of the fermi level relative to
the intrinsic fermi level.
(c) Now imagine placing these three segments together to make a p-i-n device.
Draw a band diagram for the unbiased p-i-n device after it has reached a
steady state (i.e. after carriers are done migrating). Be sure to label the
valence and conduction bands, the Fermi level, and the intrinsic Fermi level.
Indicate on your graph for part a) the directions in which electrons and holes
would migrate after the layers are brought together.
(d) Draw a diagram of the space charge distribution and the electron concentration
in the device after the device has reached a steady state.
(e) Draw a graph of the electric field with respect to position.
(f) Draw a graph of the voltage with respect to position.
65

(g) What is the built-in voltage for this cell?


(h) p-i-n structures with wide intrinsic layers are used in thin-film silicon solar cells
because of the poor diffusivity of carriers in amorphous and microcrystalline
silicon. How might using a p-i-n structure improve the efficiency of these solar
cells? Hint: See part (e).
Solution:
(a) The band diagram is shown in figure 4.13.

Figure 4.13: Band diagram of p-type, intrinsic and n-type regions, before assembly into
p-i-n diode.
(b) We can find the differences in Fermi levels by using the relationship between
carrier concentration and Fermi level. In the n-type region we have:
nn = Nc e(Ec n )/kB T
where Nc is the conduction band effective density of states, Ec is the conduction band edge, and n is the Fermi level in the n-type region. In the intrinsic
region we have:
ni = Nc e(Ec i )/kB T
66

Taking the ratio of these and solving for the Fermi level difference we find:
 
nn
n i = kB T ln
ni
 18 
10
eV
= 0.026 ln
1010
= 0.48 eV
where we have used ni = 1010 cm3 Considering the p-doped region we similarly have:
pp = Nv e(p Ev )/kB T
and
pi = Nv e(i Ev )/kB T
Again taking the ratio and solving for the Fermi level difference we find:
 
pi
p i = kB T ln
pp
 10 
10
eV
= 0.026 ln
1015
= 0.30 eV
(c) As the device is assembled from the p-type, intrinsic, and n-type Si, electrons
from the n-type region and holes from the p-type region diffuse into the intrinsic region, setting up a built in potential that opposes further diffusion.
Eventually the Fermi levels from the three regions align. The resulting band
diagram is shown in figure 4.14. The direction of diffusion of the electrons and
holes is shown in figure 4.13.
(d) As electrons from the n-type and holes from the p-type region diffuse into the
intrinsic region they leave behind fixed charges. The resulting space charge
distribution is shown in figure 4.15.
(e) The space charge results in a field as shown in figure 4.16.
(f) The field results in a voltage profile as shown in figure 4.17.
(g) The total built in voltage for this cell is given by:
Vbi = e(n p ) = 0.78 V
67

Figure 4.14: Band diagram of p-i-n diode.

Figure 4.15: Space charge and electron distribution

68

Figure 4.16: Electric field vs. position

Figure 4.17: Voltage vs. position

69

(h) Minority carrier diffusion lengths in thin-film silicon materials are very short.
In a p-n junction, many photogenerated carriers would be unable to diffuse
to the junction to get collected. A p-i-n structure results in a field across
the entire intrinsic region, which is the majority of the material in a thin-film
silicon cell. Hence most of the light-induced carriers will be formed in the
intrinsic region, and will be moved by the field (rather than diffusion) toward
the appropriate region (electrons toward the n-type and holes toward the ptype regions). This field assisted transport can take place more rapidly than
diffusion so the likelihood of recombination is reduced, resulting in greater
efficiency.
10. (From 2011 Midterm) An ideal Si solar cell (low junction recombination so m = 1,
no series resistance and infinite shunt resistance) operating at T = 300 K is exposed
to a solar power density of Ps = 1000 W/m2 . This gives a light induced current
density of JL = 0.04 A/cm2 and an open circuit voltage of 700 mV.
(a) What is the reverse saturation current density (Js ) for this solar cell?
(b) What is the current density of the cell when the operating voltage is 616 mV?
(c) What is the efficiency for the solar cell under this condition?
We now consider a solar cell that has the same low junction recombination, so
m = 1, and has the same reverse saturation current density (Js ) and is exposed
to the same solar illumination, with the same light induced current density (JL ).
But this cell has a shunt resistance of Rsh = 50 for a solar cell with an area of
A = 1 cm2 .
(d) What is the current density for this cell when the operating voltage is 616
mV?
(e) What is the efficiency for the solar cell under this condition?
Solution:
(a) Using the expression for the current-voltage behavior of an ideal solar cell for
the open circuit condition we find:
Js = JL eeVoc /kB T = 7.0 1014 A/cm2
(b) Again using the expression for the current-voltage behavior of an ideal solar
70

cell we find:


J = JL Js exp

eV
kB T


1

= 38.4 mA/cm2
(c) The efficiency can be found by dividing the product of the current density and
voltage by the input power density:
=

JV
= 23.7%
Ps

Pretty good solar cell!


(d) The current density for this non-ideal solar cell is given by:




eV
V
J = JL Js exp
1
kB T
Rsh A
where A is the cell area. Plugging in numbers we find:
J = 26.1 mA/cm2
(e) The efficiency for this cell under this condition is:
=

JV
= 16.1%
Ps

Which is quite a bit less. Interestingly the open circuit voltage only decreased
a little from 700 mV to 689 mV.
11. (From 2012 Final) We derived the current voltage behavior of a single junction cell
in class. Here we consider the current-voltage behavior of a two-junction, tandem
cell. Here we have two different cells in series, so the current running through both
is the same, and each cell contributes their voltage. So the current densities follow:
J1 = J2 J
where J1 and J2 are the net current densities through the junction 1 and 2 respectively, and J is the total current density. We assume ideal diode behavior for each
cell, so the current densities for the two cells follow:


J1 = JL1 JS1 eeV1 /kB T 1
and J2 = JL2 JS2 eeV2 /kB T 1
71

where JL1 and JL2 are the light-induced currents for the two cells, JS1 and JS2 are
the reverse bias saturation current densities for both cells, and V1 and V2 are the
voltages across both cells. The total voltage across the tandem cell will be:
V = V1 + V2
Recalling that the reverse saturation current density is proportional to the square
of the intrinsic carrier concentration, we make the assumption:
JS1 = JS0 eEg1 /kB T

and JS2 = JS0 eEg2 /kB T

where JS0 is a constant and Eg1 and Eg2 are the band gaps for the two cells. Lastly,
we design the cell with the band gaps chose so that the light-induced current is the
same for both cells, and is equal to half the light-induced current for the lower band
gap cell operating as a single junction cell. Mathematically: JL1 = JL2 = JL /2,
where JL is the light-induced current for the lower band gap cell operating as a
single junction cell
(a) Write down the equation for the total voltage for the tandem solar cell in
terms of JL , J, JS0 , Vth = kB T /e, Eg1 and Eg2 . Note, you will have to make
an approximation to get the answer in the answer sheet.
(b) Invert your expression to find the current density J as a function of JL , JS0 ,
Eg1 , Eg2 and V .
(c) If we choose to use Si for the low band gap cell (recall the band gap for Si is
1.12 eV), what is the approximate band gap for the upper cell that would result
in the same light-induced current for both cells when the device is exposed
to AM 1.5. Hint: You can get this answer from the solar photon flux plot in
Unit 1.
Solution:
(a) We start by inverting the expressions given above for the current densities to
find:




JL2 + JS2 J
JL1 + JS1 J
and V2 = Vth ln
V1 = Vth ln
JS1
JS2
We recognize that at appreciable operating voltages the numerator of the ln
72

term will have to be much larger than the denominator, so we approximate:


 



JL /2 J
JL /2 J
V = V1 + V2 = Vth ln
+ ln
JS1
JS2
= Vth [2 ln (JL /2 J) ln (JS1 JS2 )]




Eg1 + Eg2
JL /2 J
+
= Vth 2 ln
JS0
kB T


Eg1 + Eg2
JL /2 J
=
+ 2Vth ln
e
JS0
(b) Inverting this we find:


JL
V (Eg1 + Eg2 ) /e
J=
JS0 exp
2
2Vth
Figure 4.18 shows plots of current and power are shown for single-junction
and tandem cells.
(c) The plot from Unit 1 is shown below in Figure 4.19 with the lines used to find
the value of Vg2 = 1.75 eV.
12. (From 2013 Final Exam) A single-junction Si solar cell is exposed to AM1.5 at
T = 300 K and a open circuit voltage of Voc = 700 mV is achieved. Calculate the
reverse bias saturation current Js . Assume ideal diode behavior and take the Si
band gap to be Eg = 1.12 eV. Your answer should be in A/cm2 .
Solution:
Using our expression for the open circuit voltage


JL
kB T
ln 1 +
Voc =
e
Ls
we find:

JL
exp (eVoc /kB T ) 1
We have calculated that for a Si band gap of 1.12 eV, AM1.5 produces a light
current of:
JL = 2.5 1017 e/(cm2 s) = 0.040 A/cm2
Js =

Inserting this we find:


Js = 7.13 1014 A/cm2
73

Current Density (A/m )

tandem
single

400
200
0
-200
0.0

0.5

1.0

1.5

2.0

Power Density (W/m )

Cell Voltage (V)


400
300

35%
23.5%

200

tandem
single

100
0
0.0

0.5

1.0

1.5

2.0

Cell Voltage (V)

Figure 4.18: The current density and power density for tandem and single junction cells.
The single junction cell has a band gap of 1.12 eV and the tandem cell has cells with
band gaps of 1.12 eV and 1.75 eV. The percentages listed on the power density plot are
the solar cell efficiencies. For these plots JS0 was that for a single-junction solar cell with
a 1.12 eV band gap and a open circuit voltage of 0.7 V.

74

4
AM1.5
5960 K Spectra

x10

1.0

1
0
0

1.5

21

2.0
x10

21

0.5

0.0

Figure 4.19: The photon flux as a function of energy. Shown also is the integrated photon
flux above the Si band gap energy of 1.12 eV.

75

76

Chapter 5
Electrochemistry and Batteries
1. You are trapped on a desert island with a radio but no batteries to run it. You
remember that you could construct a basic battery to generate voltage and power
the radio. However, you only have some iron nails and silver jewelry to work with.
(a) Draw the cell you could construct with the radio as the load. Mark the
cathode, anode, electron flow and cation/anion flow. (assume you have an
appropriate electrolyte available).
(b) Determine the half cell reactions and the full cell reaction for this cell. (ignore
the electrolyte)
(c) Determine the standard potential and the Gibbs free energy for the cell reaction.
Solution:
(a) The drawing is shown in figure 5.1.
(b) By examination of the half-cell potentials in the notes we see that the potential
for oxidization of Fe is greater than that for oxidization of Ag. In fact, the
potential for oxidization of Ag is negative, indicating that reduction of Ag is
favorable. Hence we write the half-cell reactions with Fe as the anode, where it
is oxidized and Ag as the cathode, where it is reduced. The half-cell reactions
are:
Fe

Fe2+ + 2e E = 0.44 V
+

2Ag + 2e

2Ag
E = 0.80 V
77

/0%1-2('"3(4"
8,)6("9(,)"

!"

&'6('"3(4"
+,5('"3(4"

&*"+,-.()%"

$%"&'()%"

#"

/0%1-2(07-%"

Figure 5.1: Schematic of Fe-Ag electrolytic cell.


So the total cell reaction is
Fe + 2Ag+
2Ag + Fe2+ E = 1.24 V
(c) From above we see that the standard potential is E = 1.24 V. The standard
state free energy can be found from:
G = nF E = 239 kJ/mol
2. I place 20 grams of NaCl in 56 grams of water. Calculate the atomic fraction,
molarity and molality for the sodium.
Note: To calculate the molarity you will need the volume of the solution. We can
find this by using the partial molar volumes for H2 O and NaCl in solution through:
V = nNaCl VNaCl + nH2 O VH2 O
where ni and and Vi are s the number of moles the molar volume of i. Use:
VH2 O = 18 ml/mol

and

VNaCl = 16.7 ml/mol

You might also want to know the atomic masses:


78

element
H
M (g/mole) 1

O Na
Cl
16 23 35.45

Solution: To start we calculate the number of moles of each constituent:


NNa = NCl =

20 g
= 0.34 mol
23 g/mol + 35.45 g/mol

and
NH2 O =

56 g
= 3.11 mol
18 g/mol

The atomic fraction is:


xNa =

0.34 mol
= 0.09
3.11 mol + 2 0.034 mol

The molality is the number of moles per mass so we have:


mNa =

0.34 mol
= 4.47 mol/kg
0.02 kg + 0.056 kg

To find the molarity we need to calculated the volume of the solution. Using the
equation and information given in the problem statement we find
V

= nNaCl VNaCl + nH2 O VH2 O


= 0.34 mol 16.7 ml/mol + 3.11 mol 18 ml/mol = 61.7 ml

The molarity is then:


MNa =

0.34 mol
= 5.5 mol/L
0.062 L

3. You are making a battery out of Ti and Mn.


(a) Write out the balanced electrochemical reaction.
(b) What is the potential of the cell according to the half-cell reaction chart?
(c) Calculate the gram-equivalent for each material.
(d) What is the theoretical capacity of the cell, in Ah/g? Ignore the masses of the
electrolyte, case, etc.
(e) What is the specific energy available from the cell, in J/g?
79

Solution:
(a) The half cell reactions are:
Ti

Ti2+ + 2e E = 1.75 V

Mn + 2e

Mn
E = 1.05 V
2+

So the total cell reaction is


Ti + Mn2+
Ti2+ + Mn E = 0.70 V
(b) From above see the standard potential is E = 0.70 V.
(c) The gram equivalent of each material is found from the atomic mass divided
by the number of electrons gained or lost in the reaction. For Ti we find:
eq
gTi
=

47.88 g/mol
= 23.94 g/mol eq
2

eq
gMn
=

54.94 g/mol
= 27.47 g/mol eq
2

and for Mn:

(d) To find the theoretical capacity we first find the number of Ah/g for each of
the electrodes:

Ah
Fn
26.8 Ah/mol
=
=
= 1.12 Ah/g

g Ti MA
23.94 g/mol eq
For Mn we find:

Ah
Fn
26.8 Ah/mol
= 0.98 Ah/g
=
=

g Mn MA
27.47 g/mol eq
The theoretical capacity for the cell is then:


1
Ah
1
1
=
+
= 0.52 Ah/g
g cell
1.12 Ah/g 0.98 Ah/g
(e) The specific energy available from the cell can be found by multiplying the
voltage by the theoretical capacity:
Energy
= 0.70 V 0.52 Ah/g 3600 s/h = 1.32 kJ/g
g
80

4. The cell reactions in a Li ion battery are given by:


anode
Lix C6 xLi+ + xe + 6C
cathode
Li1y CoO2 + yLi+ + ye LiCoO2
cell y(Lix C6 ) + x(Li1y CoO2 ) 6yC + x(LiCoO2 )
The anode has a capacity of 0.34 Ah/g and the cathode has a capacity of 0.137
Ah/g.
(a) Calculate the cell capacity (Ah/g).
(b) Find x and y.
(c) If the cell operates at 3.8 volts, calculate the energy mass density (Wh/g).
Solution:
(a) The cell capacity is found from:

1
1
Ah
1
=
+
g
0.34 Ah/g 0.137 Ah/g
= 0.098 Ah/g
(b) Beginning with the anode, we note that there are x unit charges for each mole
of Lix C6 . So the anode capacity is given by:
cA =

xF
xF
=
MLix C6
xMLi + 6MC

where I have used a shorthand symbol cA for the anode capacity in Ah/g.
Solving for x we find:
6cA MC
x=
F cA MLi
Inserting numbers we find
x=1
Turning to the cathode, we note that for there is y unit charges for each mole
of LiCOO2 . Thus the cathode capacity is given by:
cC =
81

yF
MLiCoO2

where I have used a shorthand symbol cC for the cathode capacity in Ah/g.
Solving for y we find:
MLiCoO2 cC
y=
F
Plugging in numbers we find
y = 0.5
(c) The power density is just the cell capacity times the voltage. We find
power density = 0.098 Ah/g 3.8 V = 0.37 Wh/g
5. (From the Final Exam in 2007)
(a) Using the table for standard electrode potential from Lecture 15, slide 7,
calculate the standard potential E and the standard free energy G for
the reaction:
1
H2 + O2
H2 O
(5.1)
2
(b) What are E and G for the reaction:
2H2 + O2
2H2 O
We are interested in making a hydrogen-oxygen fuel cell from reaction 5.1. However
we want a higher voltage. In a flash of brilliance we realize that cranking up the
pressure of the hydrogen and oxygen will drive the reaction more to the right and
thus should give a higher cell voltage.
(c) Calculate the theoretical cell potential we will get if we operate the cell using
pure H2 at a pressure of 10 atmospheres, and air at a pressure of 5 atmospheres.
The cell is operated at a temperature below 100 C so the water is liquid. Take
air to be 21% O2 .
Solution:
(a) The reaction above can be found by summing the reactions:
1
2

(O2 + 4H+ + 4e
2H2 O) E = 1.23 V
2 (H+ + e
21 H2 )
E = 0.00

Thus we find
E = 1.23 eV
82

since the standard potential is not dependent on the amount of material in


the reaction. Using the standard state Nernst equation we find the reaction
free energy:
G = nF E
= 2 96500 C/mole 1.23 V
= 237.4 kJ/mole
(b) The reaction above can be found by summing the reactions:
O2 + 4H+ + 4e
2H2 O E = 1.23 V
4 (H+ + e
12 H2 ) E = 0.00
Thus we find
E = 1.23 eV
same as before. Again, the standard potential is not dependent on the amount
of material in the reaction. Using the standard state Nernst equation we find
the reaction free energy:
G = nF E
= 4 96500 C/mole 1.23 V
= 474.7 kJ/mole
(c) Here we use the Nernst equation for this reaction:
RT
aH 2 O
ln

nF
aH2 aO2
1
RT
p
ln
= E
nF
PH2 PO2
8.314 J/(mole K) 300 K
1

= 1.23 V
ln
2 96500 C/mole
10 0.21 5
= 1.26 V

E = E

So not much change for all the work in increasing the pressure.
6. (From the Final Exam in 2007) The measured activation potential versus current
for a battery is shown in Figure 5.2.
(a) From the plot in Figure 5.2 find the exchange current density j0 .
83

Figure 5.2: Measured activation potential versus current density for hypothetical cell.
Also shown is the fit to the Tafel equation (dashed line).
(b) Also from the plot, find the Tafel slope and calculate . (Assume there are
two electrons transfered in the cell reaction and that the measurement is done
at T = 300 K.)
(c) What is the activation potential when we drive this cell at a current density
of j = 0.5 A/cm2 ?
Solution:
(a) The exchange current density is the forward current when the activation potential act = 0. This corresponds to the horizontal axis intercept of the
extrapolation of the Tafel equation (act = a + b ln j) fit to the high activation
potential behavior. In this case we find:
J0 = 0.05 mA/cm2
(b) We can estimate the slope of the Tafel equation fit by using the points:
j (mA/cm2 )
0.1
1.0
84

act (V)
0.038
0.158

We find:
1 2
ln(j1 /j2 )
0.12 V
=
ln(10)
= 0.052 V

b =

From the notes:


b=

RT
nF

So
RT
bnF
= 0.25

(c) We can now use the Tafel equation to find the activation potential at a given
current, so long as it is in the high-bias regime. From the plot we see that this
is indeed the case: the Tafel does a good fit to the data well before we get to
the given current density. Rearranging the Tafel equation a bit we find:
act = b ln

j
j0

= 0.051 V ln

500
0.05

= 0.48 V
7. (From the Final Exam in 2007) Your brand new Honda FCX Clarity is powered by a
hydrogen fuel cell. The electric motor draws 100 kW at full power and requires 1000
V to operate. The hydrogen fuel cell operates off the reaction 2H2 + O2
2H2 O
with a full cell reaction potential of 1.23 V.
(a) How many fuell cells do you need to stack together to achieve the voltage
required by the electric motor?
(b) What current (in Amps) do you need from your fuel cell stack to feed your
electric motor at full power?
(c) What is the flow rate of H2 molecules (in moles H2 /sec) required to power the
fuel cell stack at full power?
85

(d) If the maximum proton exchange rate through the membrane in this fuel cell
is jH+ = 1 105 moles/cm2 s, how large an area must each fuel cell in the
stack have?
(e) What is the flow rate of hydrogen required to supply the fuel cell stack at full
power (in L/s)?
(f) At highway speeds (55 mph) the car only used 24 kW of power. If the manufacturer wants the driver to be able to go 300 miles at highway speeds on a
full tank, how much hydrogen gas (in L at STP) does the fuel tank need to
hold? A mole of gas at STP occupies 22.4 L.
Solution:
(a) To get a total voltage of 1000 V, you will need Ncell = 1000/1.23 = 813 cells.
(b) The current is given by
P
V
1 105 W
=
1000 V
= 100 A

I =

(c) Each H2 molecule contributes 2 electrons to the current to each cell. Therefore
the total hydrogen flow rate FH2 is given by:
INcell
2F
100 A 813
=
2 96500 C/mole
= 0.42 mole/s

F H2 =

(d) The total number of protons going through each cell must add up to the cell
current. Therefore the area is given by:
A =

I
nF jH+

100 A
96500 C/mole 105 moles/cm2
= 103.6 cm2
=

86

(e) Since each mole occupies 22.4 L, the volume flow rate is given by 22.4 L/mole
0.42 mole/s = 9.4 L/s
(f) From above we note that we have calculated that it the fuel cell requires 9.4
L at STP to produce 100 kJ of energy. The to travel 300 miles, the energy
(E300 ) required is:
3600 s
300 miles

55 miles/hour
hour
5
= 4.71 10 kJ

E300 = 24000 W

Therefore the volume is given by:


9.4 L
4.71 105 kJ = 44300 L
100 kJ
This would be a cubic box about 12 feet on a side.
V =

8. (From 2008 Midterm) A hydrogen-oxygen PEM fuel cell operates with the following
properties 1 :
PO2
1 atm

PH 2
1 atm

Temp.
350 K

j0 (H2 )
0.10 A/cm2

j0 (O2 )
4

10

A/cm

(H2 )
0.50

(O2 )
h/
0.30 0.01 cm2

(a) What is the theoretical or open circuit voltage for this cell?
(b) What is the cell voltage when the cell operates with a current density of
500 mA/cm2 .
(c) What is the efficiency of the cell when it operates with a voltage of 800 mV?
Solution:
(a) Since the cell is operating at atmospheric pressure for both the O2 and H2
gasses, the theoretical open circuit voltage of the cell is just the potential of
the reaction. Using the table from the notes we find
E = E = 1.23 V
(b) The polarization loss for these operating conditions can be found by:


RT ( ln j0 (O2 ) + ln j) ( ln j0 (H2 ) + ln j)
jh RT
jL
cell =
+
+
+
ln
nF
(O2 )
(H2 )

nF
jL j
1

Adapted from table 6.1 in Fuel Cell Fundamentals, by OHayre et al.

87

jL
2 A/cm2

where we have included the activation potential for both the hydrogen electrode (anode) and the oxygen electrode (cathode). Inserting numbers we find:
cell = 0.486 V
Hence the cell potential is:
E = 1.23 V 0.486 V = 0.74 V
(c) Since the efficiency is the ratio of the output and input power, and the input
power is just the free energy of the reaction times the reaction rate, we find:
(efficiency) =

800 mV
Vj
=
= 65%

E j
1230 mV

9. (From 2008 Midterm) In a direct methanol fuel cell (DMFC) water and methanol
(CH3 OH) are oxidized at the anode with the half cell reaction:
CH3 OH + H2 O CO2 + 6H+ + 6e
The reaction products for the total cell reaction are CO2 and H2 O. The total cell
reaction also has a standard free energy of reaction
G = 700 kJ/mol
(a) What is the half-cell reaction at the cathode (where oxygen and the H+ ions
are reduced)?
(b) Write down the balanced total cell reaction?
(c) What is the standard potential for this cell?
Solution:
(a) To find the cathode half cell reaction we realize that the reactants are O2 , H+
and electrons combining to form water. Thus we have:
mO2 + 6H+ + 6e nH2 O
Balancing the number of each species on both sides of the reaction we find:
2m = n

and

2n = 6

Solving we find m = 3/2 and n = 3. So the half reaction is:


3
O2 + 6H+ + 6e 3H2 O
2
88

(b) The balanced equation for the total cell reaction is then the sum of the two
half reaction equations:
CH3 OH + H2 O CO2 + 6H+ + 6e
+ 6H+ + 6e 3H2 O
CH3 OH + 23 O2 CO2 + 2H2 O

3
O
2 2

(c) The standard potential for this cell is found from the Nernst equation:
E

G
700 kJ/mol
=
= 1.21 V
nF
6 96500 C/mol

10. (From 2009 Midterm) We construct a battery with an electrode resistance of


Relect = 0.02 and an electrolyte conductivity of electrolyte = 0.10 (cm)1 . The
cell has area 10 cm2 and electrolyte thickness 1 mm.
(a) Assuming that the electrode and electrolyte resistances are the only contributions to the cell resistance, what is the resistance of this cell?
(b) If we operate the cell at a current density of j = 0.50 A/cm2 , what is the drop
in cell potential due to this resistance?
(c) What is the power dissipated by this resistance?
Solution:
(a) The resistance of the cell is found from:
R = Relect +

h
A

where is the conductivity of the electrolyte, A is the cell area, and h is the
electrolyte thickness. Inserting numbers we find:
R = 0.12
(b) The cell current will be:
I = JA = 0.50 A/cm2 10 cm2 = 5 A
The potential drop is then:
= IR = 0.6 V
89

(c) The power dissipated by this resistance is the current times the potential drop:
P = I = 5 A 0.6 V = 3 W
11. (From 2009 Midterm) A hydrogen-oxygen solid oxide fuel cell operates with the
following properties2 :
PO2 PH2 PH2 O
atm atm atm
5
4
1

Temp.
K
1000

j0 (H2 )
A/cm2
10

j0 (O2 )
A/cm2
0.10

(H2 )

(O2 )

0.50

0.30

h/
cm2
0.01

jL
A/cm2
2

(a) Write down the balanced half cell reactions and cell reaction for this fuel cell.
(b) What is the theoretical or open circuit voltage for this cell? Note that at the
operating temperature of 1000 K the water will be a gas.
(c) What is the total polarization loss when the cell operates with a current density
of 1.5 A/cm2 .
Solution:
(a) The half cell and cell reactions are:
2H2 (g) 4H+ + 4e E = 0.0 V
O2 (g) + 4H+ + 4e 2H2 O
E = 1.23 V
O2 (g) + 2H2 (g) 2H2 O
E = 1.23 V
(b) We can find the theoretical potential using the Nernst Equation:
!
RT
aH 2 O
Vth = E
ln
1/2
nF
aH2 aO2
!
RT
PH 2 O
= E
ln
1/2
nF
PH 2 P O
2

Inserting numbers we find:


Vth = 1.32 V
(c) The polarization loss for these operating conditions can be found by:


RT ( ln j0 (O2 ) + ln j) ( ln j0 (H2 ) + ln j)
jh RT
jL
cell =
+
+
+
ln
nF
(O2 )
(H2 )

nF
jL j
2

Adapted from table 6.1 in Fuel Cell Fundamentals, by OHayre et al.

90

where we have included the activation potential for both the hydrogen electrode (anode) and the oxygen electrode (cathode). We note that for the hydrogen electrode, j < l0 . Hence, the Tafel equation is not valid for that
electrode (recall that the Tafel equation was valid only for large current densities (j  j0 ). So for this electrode we would have to use the full Butler-Volmer
equation to find the potential associated with the given current density. However, it is very likely that the potential drop at this interface is negligibly
small, so we take it to be zero. Inserting numbers we find:
cell = 0.46 V
Note: For the purposes of this exam, a wide variety of answers was accepted,
including those which incorporated the negative hydrogen electrode potential
predicted by the Tafel equation, a negative sign error in the activation potential that had infected the notes, and using n = 4, since the course did not give
a good explanation of how to determine the correct value of the number of
electrons in the reaction.
12. A diver goes to a depth of 100 feet, where the pressure increase is equal to the weight
per area of water above the diver. At that depth his blood becomes saturated with
N2 from the air he is breathing at that pressure. We are going to calculate the
amount of nitrogen that will come out his 5 liters of blood as he ascends to the
surface. You may or may not need the following information:
The density of water is 1 kg/liter. Air is 80 atomic % N2 . The solubility of nitrogen
in water at 25 C in equilibrium with one atmosphere of N2 is 6.25 104 moles/kg.
The divers blood temperature is 98.6 F. We will assume that gaseous N2 behaves
like an ideal gas at all pressures encountered. A mole of an ideal gas occupies 22.4
liters at STP.
(a) What is the free energy for nitrogen in the reference state of unit molality
(G
N2 (aq) )?
(b) What is the partial pressure of N2 at the depth of 100 feet.
(c) Use the information given in this problem to calculate the pressure at the
100 foot depth. (Dont forget that there is one atmosphere pressure at the
surface.)
(d) What is the molality of N2 in the divers blood at the depth of 100 feet.
91

(e) What is the volume at STP of the excess N2 gas dissolved in the divers blood.
(STP refers to standard temperature and pressure, which is 298 K and 1
atmosphere.)
Solution:
(a) The standard-state free energy can be found from the equilibrium condition
given in the problem statement. The free energy of the gas phase N2 is given
by:
GN2 (g) = RT ln P
Since the standard state free energy of formation for N2 in the gas phase is
zero. The free energy of the N2 dissolved in the aqueous phase is:
GN2 (aq) = G
N2 (aq) + RT ln m
Setting GN2 (g) = GN2 (aq) and taking P = 1 atm, and m = 6.25 104 , we find:
G
N2 (aq) = 18.3 kJ/mole
(b) It is easier to do part 12c first. The partial pressure of N2 will be given by
PN2 = 0.8 Ptotal = 3.16 Atm
(c) The total pressure at a depth of 100 feet is given by:
Ptotal = 1 Atm + hg
where is the density of the water (taken here to be 1 g/cm3 ), g is the
acceleration due to gravity, given by g = 9.8 m/s2 , and h is the height of the
water column. We find:
Ptotal = 3.95 Atm
(d) The molality of the divers blood can be found from the equilibrium condition.
Again setting the free energies of the N2 gas and aqueous phases equal we find:
RT ln PN2 = G
N2 (aq) + RT ln m
Solving for m we find:
m = PN2 exp
92

G
N2 (aq)
RT

For the conditions when the diver is at depth (PN2 = 3.16 atm and T =
98.6 F = 310 K) we find
m = 2.6 103 mol/kg
To find the excess N2 for part 12e we need to also calculate the molality when
the diver is at the surface (PN2 = 0.8 atm and T = 98.6 F = 310 K). For this
case we find
m = 5.0 104 mol/kg
(e) The total number of moles of excess N2 is just the mass of blood times the
difference in molality.

nN2 = 5 kg 2.6 103 moles/kg 5.0 104 moles/kg
= 0.011 moles
The volume at STP will be 24.5 Liters times the number of moles:
VN2 = 24.5 L/mole 0.011 mole = 260 ml
Note: The more common STP condition is at T = 0 C = 298.15 K. This gives
a molar volume of 22.4 L/mole. Either answer is ok here.
13. (From 2010 Midterm) In a lead-acid battery the anode and cathode reactions are
given by:

Anode : Pb(s) + SO2


4 (aq) PbSO4 (s) + 2e

Cathode : PbO2 (s) + 4H+ (aq) + SO2


4 (aq) + 2e PbSO4 (s) + 2H2 O(l)

(a) What is the full cell reaction for this battery?


(b) The standard state fee energy for the full reaction is
G = 393.7 kJ/mol.
What is the standard potential for this cell?
(c) Using the table of atomic weights below, what are the theoretical capacities,
in Ah/g, for each electrode?
Pb
S
O H
Element
Mass
207.2 32.1 16 1
93

(d) What is the capacity for the lead acid cell?


(e) In a homework problem we showed that a typical lithium ion battery, with a
graphite anode and Lix CoO2 cathode had a similar capacity of 0.098 Ah/g.
Compared to lead acid batteries, what advantages do Li ion batteries have
that are leading them to be used for a variety of mobile applications?
Solution:
(a) The full cell reaction is:
Pb(s) + PbO2 (s) + 4H+ (aq) + 2SO2
4 (aq) 2PbSO4 (s) + 2H2 O(l)
(b) The standard potential can be found from the standard state Nernst Equation:
G
393.7 kJ/mol
E =
=
= 2.04 V
nF
2 96.5 kC/mol

(c) The anode reaction uses one mole of Pb, one mole of S, and four mols of O to
make two moles of electrons. Hence the capacity of the anode is:

nF
Ah
=

g A
MPb + MS + 4MO
2 26.8 Ah
=
(207.2 + 32.1 + 4 16) g
= 0.177 Ah/g
The cathode uses one mole of Pb, one mole of S, six moles of O, and four
moles of H. The capacity of the cathode is then:

Ah
nF
=

g C
MPb + MS + 4MO
2 26.8 Ah
=
(207.2 + 32.1 + 6 16 + 4) g
= 0.158 Ah/g
(d) The capacity of the whole cell is then:


1
Ah
g 1  g 1
=
+


g Cell
Ah C
Ah A
= 0.083 Ah/g
94

This can also be found from the cell reaction, knowing that there are two
moles of electrons involved.
14. (From 2010 Midterm) A direct methanol fuel cell (DMFC) uses methanol Ch3 OH
and oxygen as fuel. The half cell and full reactions are given below.
Anode : CH3 OH + H2 O CO2 + 6H+ + 6e
Cathode :

3
O2 + 6H+ + 6e 3H2 O
2

3
Cell : CH3 OH + O2 CO2 + 2H2 O
2
In a previous problem it was determined that the standard potential for this cell
was E = 1.21 V.
(a) What is the maximum energy per mass of fuel that can be achieved for this
type of fuel cell? Ignore the mass of the oxygen, since we are assuming it
comes from air, which is 20 atomic percent oxygen. The mass of carbon is 12
g/mol. Your answer should be in kJ/g.
One problem with DMFC fuel cells is that the methanol fuel can diffuse through
the nafion electrolyte membrane from the anode to the cathode where it is oxidized. Thus a fraction of the fuel is lost. This diffusion loss is proportional to the
concentration of the methanol water mixture used to fuel the anode. To minimize
this loss, the methanol is mixed with water. Here assume that the methanol-water
solution is ideal, so the activities are given by the atomic fractions. Also assume
ideal gas behavior so the gas activity is just the partial pressure in atmospheres
and assume the activity of the product water is unity.
(b) Find the change from the standard potential for this cell when the fuel is
provided to the anode as a water-methanol mixture containing mole fraction
x of CH3 OH. Your answer should be in terms of temperature, x, the partial
pressure of oxygen and the partial pressure of CO2 .
(c) Find the cell potential in volts when we use air for the oxygen, the partial
pressure of CO2 is 0.1 atom and x = 0.1. The temperature is 300 K.
Solution:
95

(a) The current capacity per mole of reaction is:


Ah
nF
=
g
MCH3 OH
6 96.5 kC/mol
=
(12 + 4 + 16) g/mol
= 18.1 kC/g
To find the energy we multiply by the voltage:
EnergyDensity = 18.1 kC/g 1.21 V = 21.9 kJ/g
(b) From the Nernst equation we find:
kB T
ln
E =
e

PCO2

3/2

xPO2

(c) Plugging in numbers we find:


E = E + E = (1.21 0.062) V = 1.15 V
15. (From 2011 Midterm) A hydrogen anode (oxidization) reaction at T = 300 K
involving reactants at fixed surface concentration and with negligible electrolyte
and electrode resistance has a Tafel plot given by figure 5.3.
(a) Write down the equation for the reaction occurring at this electrode. How
many electrons are involved per formula unit?
(b) Using the graph, find the Tafel parameters a and b from the Tafel equation:
act = a + b ln j
(c) Find the parameter , which partitions the decrease in the forward and reverse
reaction barriers.
(d) Find exchange current j0 .
(e) What is the current density at act = 0.19 V?
Solution:
96

0.20

0.15

0.10

0.05

0.00
0.1

6 7 8 9

Figure 5.3: Tafel plot for hydrogen electrode reaction, showing activation potential as a
function of current density.
(a) The reaction equation is:
H2 2H + + 2e
Since there are two electrons per hydrogen molecule n = 2.
(b) From the graph we find two points for act and j, which, given as (j, ) in
units of A/cm2 and V respectively, (0.1,0.13) and (3.0,0.19). Using the Tafel
equation we find:
1 2
= 0.018 V and a = 0.17 V
b=
ln(j1 /j2 )
(c) The Tafel slope is given by:
b=

RT
nF

Solving for we find:


=

RT
= 0.73
bnF

(d) The Tafel parameter a is given by:


a=

RT
ln(j0 ) = b ln(j0 )
nF

Thus
j0 = ea/b = 6.3 105 A/cm2
97

(e) At arbitrary activation potential the current density is given by the ButlerVolmer equation, given by:


cR nF /RT cP (1)nF /RT

j = j0
e
e
cR
cP
For the present case with fixed surface concentrations, this can be written as:

j = j0 enF /RT e(1)nF /RT
At an activation potential of -0.19 V, the first term in parentheses is negligible,
so we can write:
j = j0 e(1)nF /RT = 3.2 mA/cm2
This is less in magnitude than the forward current at the magnitude forward
bias. This is because the reverse barrier is larger than the forward barrier,
since > 0.5.
16. (From 2012 Final) One important aspect of battery performance we did not cover
very much is the behavior of the voltage during discharge. In Unit 11 we showed
a discharge curve that plotted the battery voltage versus time, and showed that
the battery voltage decreased as the battery was discharged. In this problem we
examine this behavior. We assume we are discharging very slowly, so that there
are none of the kinetic potential losses we explored in Unit 12. In other words, for
this slow discharge, the battery voltage is limited by the thermodynamics of the
reaction, and the voltage drop just occurs because we are using up our electrodes.
For simplicity we consider a generic battery reaction
anode
A
cathode
B
cell
A+B

C
D
C +D

We designate the state of discharge with the variable x. So at x = 0 the battery


is fully charged and at x = 1 the battery is fully discharged. At partial discharge,
x is between 0 and 1. In this case, the anode has atomic fraction A given by
xA = (1 x) and atomic fraction C given by xC = x, and the cathode has atomic
fraction B given by xB = (1 x) and atomic fraction D given by xD = x. We
assume the anode and cathode behave as ideal solutions, so the activities of the
species are equal to the atomic fraction.
(a) Write down the free energy for the battery reaction as a function of state of
discharge x.
98

(b) Calculate the cell potential for this battery as a function of state of discharge
x.
(c) Take the standard state free energy for this reaction to:
G = 500 kJ/mole
Assume that there are 2 electrons transferred in the cell reaction, and that
the cell temperature is 300 K. Graph the cell potential as a function of state
of discharge x.
Solution:
(a) The free energy for this reaction is given by:



aC aD
G = G + RT ln
aA aB


x2

= G + RT ln
(1 x)2

Answer selection:
i.

(1 x)2
G = G + RT ln
x2

x2
G = G + RT ln
(1 x)2

ii.

iii.

G = G + RT ln
iv.

G = G + nRT ln

x
1x

x
1x

(b) Using the Nernst equation, we find:


G
nF


G RT
x2
=

ln
nF
nF
(1 x)2

E =

99

Cell Potential (V)

-2.50
-2.60
-2.70
0.0

0.2

0.4

0.6

0.8

1.0

(c)
Figure 5.4: Cell potential as a function of state of discharge.
17. (From 2013 Final Exam) Recently a light powered by gravity has been gaining
attention as an option for off-grid lighting. A falling weight pulls a cable to drive a
generator, producing power for a LED light (Fig.5.5). Here we examine this source
of power for lighting and compare it to other available options for off-grid lighting.
(a) What is the potential energy increase (in Joules) when a 5 kg weight is lifted
1.5 meters off the floor?
(b) What is the maximum power that can be generated by letting the 5 kg weight
fall the 1.5 meters back to the floor in 15 minutes?
(c) Assuming the human is 15% efficient in their use of food (chemical energy),
how much extra chemical energy must be consumed by the human to power
this light for 4 hours of operation?
A competing technology uses a photovoltaic cell to charge a battery which can
then be used to power a LED light. To compare these technologies, we make some
reasonable assumptions. We assume the solar cell is a square 10 cm on a side, and
that it has an efficiency of 15%. We assume the battery is a LaNi5 nickel metal
hydride rechargeable battery with a mass of 50 grams, and we assume that the
battery has an actual specific charge of 50% of its theoretical specific charge. We
also assume that the battery produces an average voltage of 1.1 V.
(d) How much power will be generated by our solar cell if we place it in 1000 W/m2
direct incident sunlight?
(e) How much energy can the solar cell produce during a day that receives the
100

LED'Light'

Generator'

Weight'

Figure 5.5: Schematic of gravity-powered generator and LED light. When the weight
gets to the bottom, it must be lifted to begin the power generation again. In this sense
it is a human powered light.

101

equivalent light of 4 hours of 1000 W/m2 direct incident sunlight?


(f) How much energy can the battery store?
Solution:
(a) The increase in potential energy is given by:
E = mgh
= 5 kg 9.8 m/s2 1.5 m
= 73.5 J
(b) The power is given by:
E
t
73.5 J
=
900 s
= 0.08 W

Pweight =

This will power only a very dim LED bulb.


(c) To get 4 hours of operation we must lift the weight 16 times. The total
potential energy which must be imparted to the weight to get 4 hours of run
time is:
Epot = 16 E = 1.2 kJ
To produce that much energy a human must consume food energy equal to:
Efood =

Epot
= 7.8 kJ
0.15

(d) The solar cell will produce a power of:


PPV = 1000 W/m2 (0.10 m)2 0.15 = 1.5 W
(e) The energy produced by our solar cell is:
EPV = 1.5 W 4 3600 s = 21.6 kJ
Thus if our battery can store this power, we can drive a 1.5 W LED bulb for
4 hours with the energy we have produced with our solar cell. This is a fairly
bright bulb and is over 18 times the power of the gravity powered light.
102

(f) From our lecture notes, a LaNi5 nickel metal hydride battery has a theoretical
specific charge of 0.16 Ah/g. Our 50 g battery can then store energy:
Ebattery = 0.16 Ah/g 0.5 50 g 1.1 V = 15.8 kJ
So our solar cell charged battery powered LED light can produce way more
light than the gravity powered light.
18. (From 2013 Final Exam) In the TV show Breaking Bad the characters attempt
to use HF acid to dissolve guns (among other things). Here we consider instead
dissolving iron with sulfuric acid.
(a) Write the balanced equation for reacting iron in dilute sulfuric acid to form
aqueous FeSO4 .
(b) How many liters of 1 molar (1 mole per liter) sulfuric acid would be required
to dissolve 1 kg of iron? Assume the reaction from the previous part goes to
completion. The molecular mass of Fe is 55.85 g/mol.
Today, though, there are too many guns and not enough acid. Instead all they
have are large tanks filled with excess ZnSO4 in solution (leftover from a previous
episodes events) as well as excess aluminum foil. They dip the foil and the guns
into a tank such that electrons can flow between the two metals outside of solution.
(c) Identify the anode material and cathode material in the resulting electrochemical reaction.
It is apparent that they must use an outside source of electrical power to drive the
dissolution of the iron. They use a small current so that the dissolution proceeds
with the minimum voltage required.
(d) How much electrical energy supplied this way is thus required to dissolve an
additional 1 kg of iron?
(e) The characters realize that their hideout has been discovered by the police
and they still have one last handgun that weighs 0.25 kg to dissolve. The cops
will get there in an hour, so they have to speed up the reaction by driving it
at a higher current. What is the minimum total voltage theyll need to drive
the reaction to get rid of the gun in time? Consider excess potential due to
activation losses only, and the exchange current I0 to be 1 A for the reaction
for the entire tank (not a current density). Take to be 0.5 and the entire
hideout is at room temperature.
103

Solution:
(a) The reaction is:
H2 SO4 (aq) + Fe FeSO4 (aq) + H2
(b) The number of moles of Fe in 1 kg is:
n=

1000 g
= 17.9 moles
55.85 g/mol

Thus it will take 17.9 moles of H2 SO4 to dissolve the 1 kg of Fe. This is the
amount contained in 17.9 liters of 1 M H2 SO4 .
(c) The half cell potential for Al, at -1.66, is the lowest of all three elements. It
will lose electrons and Al3+ will come into solution. The only species available
to accept electrons is Zn2+ , which will plate onto Fe. In this sense Zn is the
cathode material, and Fe is no different from a wire connecting Zn to Al. The
cell reaction is then:
3
3
Al + Zn2+ Zn + Al3+
2
2
(d) First it is important to recognize that the reaction which dissolves iron will
be:
Fe + Zn2+ Zn + Fe2+
This reaction has a cell potential of 0.440.76 = 0.32. The reason aluminum
is not involved is again because the only ions available are Zn2+ . The energy
required is just the potential times the number of electrons exchanged in the
dissolution of 1kg iron:
E=

2 mol electrons 96500 C


1000 g

(0.32) = 1.1 MJ
55.85 g/mol
mol Fe
mol

(e) We are given all the relevant terms except for I, which can be calculated from:
I=

250 g
96500 C
2
/3600s = 240A
55.85 g/mol
mol

Then the activation potential can be calculated, assuming


act =

R 300
ln(240/1) = 0.14 V
0.5 2 F

Thus the total voltage is 0.46 V.


104

I
I0

j
:
j0

19. (From 2013 Final Exam) In the process of electrolysis, electrical power is used to
separate water into oxygen and hydrogen molecules via the reaction:
1
H2 O H2 + O2
2
This is very much like running a hydrogen fuel cell in reverse. Here we assume
the electrolysis cell has the similar operating parameters to the fuel cell in problem
8 in chapter 4 of the HW problems, except now we assume that only the activation potential for the hydrogen reaction is non-negligible. All other potentials are
negligible. Hence the relevant parameters are:3 :
PO 2
1 atm

PH 2
1 atm

Temp.
350 K

j0 (H2 )
0.10 A/cm2

(H2 )
0.50

(a) What is the minimum voltage needed to drive this reaction at these conditions?
(b) What is the current density in A/cm2 at a voltage of 1.5 V?
(c) What area of cell in cm2 do we need to get a rate of H2 production of 1
mol/sec?
Solution
(a) The minimum voltage is the standard potential of 1.23 V.
(b) The current density can be found by rearranging or equation for activation
potential:
RT
j
act =
ln
nF (H2 ) j0 (H2 )
to find:


nF 0 (H2 )act
j = j0 (H2 ) exp
RT
Plugging in numbers we find:
J = 772 A/cm2
(c) Each mole of electrons will produce a half of a mole of H2 . Therefore to get
a mole of H2 per second we need to produce two moles of electrons, or one
faraday of charge. Therefore we find the area is:
A=
3

2 mol F
= 250 cm2
s j

Adapted from table 6.1 in Fuel Cell Fundamentals, by OHayre et al.

105

Das könnte Ihnen auch gefallen